You are on page 1of 71

Chennai Mathematical Institute

BSc (Honours) Mathematics and Computer Science


Topics covered in entrance examination
The entrance examination for the B.Sc. (Hons) Maths and Computer Science program, is a
test of aptitude to do Mathematics. We expect the students to be familiar with the following
topics.
Algebra: Arithmetic and geometric progressions, arithmetic mean, geometric mean,
harmonic mean and related inequalities, polynomial equations, roots of polynomials,
matrices, determinants, linear equations, solvability of equations, binomial theorem and
multinomial theorem, permutations and combinations, mathematical induction. prime
numbers and divisibility, GCD, LCM, modular arithmetic logarithms, probability.
Geometry: Vectors, triangles, two dimensional geometry of Conics - straight lines,
parabola, hyperbola, ellipses and circles, tangents, measurement of area and volume,
co-ordinate geometry.
Trigonometry: addition, subtraction formulas, double-angle formulas.
Calculus: Limits, continuity, derivatives, integrals, indefinite and definite integrals,
maxima and minima of functions in a single variable, series and sequences, convergence
criterion.
Complex numbers, roots of unity.
Suggested reading material
The following books are suggested, in addition to the Class XI and XII Mathematics Textbooks of the National Council of Educational Research and Training, New Delhi.
1. V. Krishnamoorthy, C.R. Pranesachar, K. N. Ranganathan, B.J.Venkatachala: Challenge and Thrill of Pre-College Mathematics, New Age International Publishers.
2. M.R. Modak, S.A. Katre, V.V. Acharya: An Excursion in Mathematics, Bhaskaracharya
Pratishtan (Pune).
3. D. Fomin, S. Genkin, I. Itenberg: Mathematical Circles: Russian experience, Universities Press (Hyderabad) 1998.
4. H.S. Hall, S.R. Knight: Higher Algebra.

CMI BSc (Hons) Math. and C.S., Sample Examination

PART A
Instructions:
There are 13 questions in this part. Each question carries
4 marks. Answer all questions.
1. For a polynomial f (x), let f (n) (x) denote the nth derivative
for n 1 and f (0) (x) = f (x). Is the following true or false
? Give brief reasons.
f (n) (a) = 0, f or n = 0, 1, . . . , k (xa)(k+1) divides f (x)
2. (a) Find the value of

n
k=1 k

k! as a function of n.

(b) True or false ? Give brief reasons.


1000

1
1
=
log1000! N
k=2 logk N

3. Let p(x) be a polynomial such that when divided by (x 1)


it leaves the remainder 2 and when divided by (x 2) it
leaves the remainder 1. What is the remainder when it is
divided by (x 1)(x 2)?
4. Prove that for any fixed n > 2, among the two integers
2n 1 and 2n + 1, at the most one of them can be a prime.
)n , n 1.
5. Show that n! ( n+1
2
6. Show that the sum

k! = m2

k=1

for any integer m, for n 4.

CMI BSc (Hons) Math. and C.S., Sample Examination

7. Give an example of a subset S of the plane such that the


following property is satisfied:
There exist infinitely many lines in the plane such that the
intersection S is a line segment of any length however
large.
8. True or false ? Give brief reasons. There exist non-zero
ploynomials f (x) with integercoefficients ofany degree d
5 which vanish at x = 2, 1 + 3 and 1 + 5.
9. If the sum of 113 terms of an arithmetic progression is equal
to 6780, then find the 57th term of the arithmetic progression. Give an example of such an arithmetic progression.
10. Show that if p and p + 2 with p 5, are both primes then
the number p + 1 is always divisible by 6.
11. Show that for all real numbers a, b, c,
(a + b + c)2 3(a2 + b2 + c2 )
Further show that 3 is the smallest real number with this
property.
12. True or false ? Give brief reasons. If a, b are positive integers then the number

(a + b)n + (a b)n
is an integer for all values of n.
13. True or false ? Give brief reasons. The number of common
tangents to the circles x2 + y 2 = 1 and x2 + y 2 4x + 3 = 0
is 1.

CMI BSc (Hons) Math. and C.S., Sample Examination

Part B
Instructions:
There are 6 questions in this part. Each question carries 8
marks. Answer all questions.
1. Let A(x1, y1), B(x2, y2) and C(x3, y3 ) form a triangle with
circumcentre (0, 0). Show that ABC is an equilateral triangle if and only if
x1 + x2 + x3 = y1 + y2 + y3 = 0
2. Show the following:
(a) limx0 sin(x) log(x) = 0

USE SERIES EXPANSION

(b) limn xn! = 0


n

(c) limx (cos( x ))x = 1, where is a constant. 1infinity


3. In a jail with 100 rooms, all locked initially, 100 rioters
break in and disturb the rooms in the following way. First
one stops at all rooms and opens them all. Second rioters
stops at rooms numbered 2, 4, 6, . . . and locks open rooms,
and leaving the other rooms as they were. The third rioter stops at rooms numbered 3, 6, 9, . . . and again opens a
locked room and locks an open room, leaving others undisturbed. And this process continues. After all the 100 rioters
have left which rooms would be open?
4. (a) If two sides of a triangle are given, then show that
the area of this triangle is maximum if the sides are
perpendicular to each other.

CMI BSc (Hons) Math. and C.S., Sample Examination

(b) Construction: For any given positive real number ,


find a point D on a line AB such that the difference
AD2 BD2 = . (Note that the point D need not be
within the segment AB).
5. Show that if [x] denote the greatest integer x, then

n
n
[ ]
3
3

is a natural number divisible by 3 for all values of n.


6. Is it possible to remove one square from a 5 5 board so
that the remaining 24 squares can be covered by eight 3 1
rectangles? If yes, find all such squares.
(Hint: A domino is a 2 1 rectangle. As you may know,
if two diagonally opposite squares of an ordinary 8 8
chessboard are removed, the remaining 62 squares cannot
be covered by 31 non-overlapping dominos. The reason
being, after removing the two corners 32 squares of one
colour and 30 of the other are left. No matter how you place
a domino it will cover one white and one black square.)

Chennai Mathematical Institute


Entrance Examination for B.Sc. (Mathematics & Computer Science) May 2010
Duration: 3 hours
Maximum Score: 100
PART A
Instructions:
There are 13 questions in this part. Each question carries 4 marks.
Answer all questions.
1. Find all x [, ] such that cos 3x + cos x = 0.
2. A polynomial f (x) has integer coefficients such that f (0) and f (1) are both odd numbers. Prove that f (x) = 0 has no integer solutions.
3. Evaluate:
n nk=1 xk
x1
1x
1/x
e
(b) lim
x0
x
(a) lim

4. Show that there is no infinite arithmetic progression consisting of distinct integers all
of which are squares.
5. Find the remainder given by 389 786 when divided by 17.
6. Prove that
2
3
n
1
+
+ +
=1
0! + 1! + 2! 1! + 2! + 3!
(n 2)! + (n 1)! + n!
n!
7. If a, b, c are real numbers > 1, then show that
1
1 + loga2 b

c
a

1
1 + logb2 c

a
b

1
1 + logc2 a

b
c

=3

8. If 8 points in a plane are chosen to lie on or inside a circle of diameter 2cm then show
that the distance between some two points will be less than 1cm.
xn
xn1
+
+ + x + 1, then show that f (x) = 0 has no repeated roots.
n! (n 1)!

3
3 3
and sin x + sin y + sin z = then show that x =
10. Given cos x + cos y + cos z =
2
2

+ 2k, y = + 2 , z = + 2m for some k, , m Z.


6
6
6
9. If f (x) =


11. Using the
fact that
n is an irrational number whenever n is not a perfect square,

show that 3 + 7 + 21 is irrational.


12. In an isoceles ABC with A at the apex the height and the base are both equal to
1cm. Points D, E and F are chosen one from each side such that BDEF is a rhombus.
Find the length of the side of this rhombus.
13. If b is a real number satisfying b4 +

1
i
=
6,
find
the
value
of
b
+
b4
b

16

where i =

1.

PART B
Instructions:
There are seven questions in this part. Each question carries 8 marks.
Answer any six questions.

1. Let a1 , a2 , ..., a100 be 100 positive integers. Show that for some m, n with 1 m n
100, ni=m ai is divisible by 100.
2. In
ABC, BE is a median, and O the mid-point of BE. The line joining A and O
meets BC at D. Find the ratio AO : OD (Hint: Draw a line through E parallel to AD.)
3. (a) A computer program prints out all integers from 0 to ten thousand in base 6 using
the numerals 0,1,2,3,4 and 5. How many numerals it would have printed?
(b) A 3-digt number abc in base 6 is equal to the 3-digit number cba in base 9. Find
the digits.
4. (a) Show that the area of a right-angled triangle with all side lengths integers is an
integer divisible by 6.
(b) If all the sides and area of a triangle were rational numbers then show that the
triangle is got by pasting two right-angled triangles having the same property.
b

5. Prove that

alogb x dx > ln b where a, b > 0, b = 1.

6. Let C1 , C2 be two circles of equal radii R. If C1 passes through the centre of C2 prove

R2
that the area of the region common to them is
(4 27).
6
7. Let a1 , a2 , . . . , an and b1 , b2 , . . . , bn be two arithmetic progressions. Prove that the points
(a1 , b1 ), (a2 , b2 ), . . . , (an , bn ) are collinear.

Entrance Examination for CMI BSc (Mathematics & Computer Science) May 2011
Attempt all problems from parts A and C. Attempt any 7 problems from part B.
Part A. Choose the correct option and explain your reasoning briefly. Each problem is
worth 3 points.
1. The word MATHEMATICS consists of 11 letters. The number of distinct ways to
rearrange these letters is
(A) 11! (B) 11!
(C) 11!
(D) 11!
3
6
8
2. In a rectangle ABCD, the length BC is twice the width AB. Pick a point P on side BC
such that the lengths of AP and BC are equal. The measure of angle CPD is
(A) 75 (B) 60 (C) 45 (D) none of the above
3. The number of with 0 < 2 such that 4 sin(3 + 2) = 1 is
(A) 2 (B) 3 (C) 6 (D) none of the above
4. Given positive real numbers a1 , a2 , . . . , a2011 whose product a1 a2 a2011 is 1, what
can you say about their sum S = a1 + a2 + + a2011 ?
(A) S can be any positive number.
(B) 1 S 2011.
(C) 2011 S and S is unbounded above.
(D) 2011 S and S is bounded above.
5. A function f is defined by f (x) = ex if x < 1 and f (x) = loge (x) + ax2 + bx if x 1.
Here a and b are unknown real numbers. Can f be differentiable at x = 1?
(A) f is not differentiable at x = 1 for any a and b.
(B) There exist unique numbers a and b for which f is differentiable at x = 1.
(C) f is differentiable at x = 1 whenever a + b = e.
(D) f is differentiable at x = 1 regardless of the values of a and b.
6. The equation x2 + bx + c = 0 has nonzero real coefficients satisfying b2 > 4c. Moreover,
exactly one of b and c is irrational. Consider the solutions p and q of this equation.
(A) Both p and q must be rational.
(B) Both p and q must be irrational.
(C) One of p and q is rational and the other irrational.
(D) We cannot conclude anything about rationality of p and q unless we know b and c.
7. When does the polynomial 1 + x + + xn have x a as a factor? Here n is a positive
integer greater than 1000 and a is a real number.
(A) if and only if a = 1
(B) if and only if a = 1 and n is odd
(C) if and only if a = 1 and n is even
(D) We cannot decide unless n is known.
1

Part B. Attempt any 7 problems. Explain your reasoning. Each problem is worth 7
points.
1. In a business meeting, each person shakes hands with each other person, with the
exception of Mr. L. Since Mr. L arrives after some people have left, he shakes hands only
with those present. If the total number of handshakes is exactly 100, how many people
left the meeting before Mr. L arrived? (Nobody shakes hands with the same person more
than once.)
20
2. Show that the power of x with the largest coefficient in the polynomial (1 + 2x
is 8,
3 )
i
i.e., if we write the given polynomial as i ai x then the largest coefficient ai is a8 .

3. Show that there are infinitely many perfect squares that can be written as a sum of six
consecutive natural numbers. Find the smallest such square.
4. Let S be the set of all 5-digit numbers that contain the digits 1,3,5,7 and 9 exactly once
(in usual base 10 representation). Show that the sum of all elements of S is divisible by
11111. Find this sum.
5. It is given that the complex number i 3 is a root of the polynomial 3x4 + 10x3 +
Ax2 + Bx 30, where A and B are unknown real numbers. Find the other roots.
6. Show that there is no solid figure with exactly 11 faces such that each face is a polygon
having an odd number of sides.
7. To find the volume of a cave, we fit X, Y and Z axes such that the base of the cave is
in the XY-plane and the vertical direction is parallel to the Z-axis. The base is the region
in the XY-plane bounded by the parabola y 2 = 1 x and the Y-axis. Each cross-section
of the cave perpendicular to the X-axis is a square.
(a) Show how to write a definite integral that will calculate the volume of this cave.
(b) Evaluate this definite integral. Is it possible to evaluate it without using a formula for
indefinite integrals?
8. f (x) = x3 + x2 + cx + d, where c and d are real numbers. Prove that if c > 31 , then f
has exactly one real root.
9. A real-valued function f defined on a closed interval [a, b] has the properties that
f (a) = f (b) = 0 and f (x) = f (x) + f (x) for all x in [a, b]. Show that f (x) = 0 for all x
in [a, b].

Part C. Explain your reasoning. Each problem is worth 10 points.


1. Show that there are exactly 16 pairs of integers (x, y) such that 11x + 8y + 17 = xy.
You need not list the solutions.
2. A function g from a set X to itself satisfies g m = g n for positive integers m and n with
m > n. Here g n stands for g g g (n times). Show that g is one-to-one if and only if
g is onto. (Some of you may have seen the term one-one function instead of one-to-one
function. Both mean the same.)
3. In a quadrilateral ABCD, angles at vertices B and D are right angles. AM and CN are
respectively altitudes of the triangles ABD and CBD. See the figure below. Show that BN
= DM.

In this figure the angles ABC, ADC, AMD and CNB are right angles.

Entrance Examination for BSc Programmes at CMI, May 2012


Attempt all 5 problems in part A, each worth 6 points. Attempt 7 out of the 9 problems in part B, each
worth 10 points.
Part A. (5 problems 6 points = 30 points.) Clearly explain your entire reasoning.
1. Find the number of real solutions to the equation x = 99 sin(x).
2. A differentiable function f : R R satisfies f (1) = 2, f (2) = 3 and f (3) = 1. Show that f (x) = 0
for some x.
3. Show that

ln(12)
ln(18)

is irrational.

4. Show that
lim

x100 ln(x)
= 0.
ex tan1 ( 3 + sin x)

5. (a) n identical chocolates are to be distributed among the k students in Tinkus class. Find the
probability that Tinku gets at least one chocolate, assuming that the n chocolates are handed out one
by one in n independent steps. At each step, one chocolate is given to a randomly chosen student, with
each student having equal chance to receive it.
(b) Solve the same problem assuming instead that all distributions are equally likely. You are given
that the number of such distributions is n+k1
k1 . (Here all chocolates are considered interchangeable
but students are considered different.)
Part B. (9 problems 10 points = 90 points.) Clearly explain your entire reasoning.
Attempt at least 7 problems. You may solve only part of a problem and get partial credit. If you cannot
solve an earlier part, you may assume it and proceed to the next part. For all such partial answers,
clearly mention what you are solving and what you are assuming.

1. a) Find a polynomial p(x) with real coefficients such that p( 2 + i) = 0.


b)Find a polynomial q(x) with rational coefficients and having the smallest possible degree
such that

q( 2 + i) = 0. Show that any other polynomial with rational coefficients and having 2 + i as a root
has q(x) as a factor.
2. a) Let E, F, G and H respectively be the midpoints of the sides AB, BC, CD and DA of a convex
quadrilateral ABCD. Show that EFGH is a parallelogram whose area is half that of ABCD.
b) Let E = (0, 0), F = (0, 1), G = (1, 1), H = (1, 0). Find all points A = (p, q) in the first quadrant
such that E, F, G and H respectively are the midpoints of the sides AB, BC, CD and DA of a convex
quadrilateral ABCD.
3. a) We want to choose subsets A1 , A2 , . . . , Ak of {1, 2, . . . , n} such that any two of the chosen subsets
have nonempty intersection. Show that the size k of any such collection of subsets is at most 2n1 .
b) For n > 2 show that we can always find a collection of 2n1 subsets A1 , A2 , . . . of {1, 2, . . . , n} such
that any two of the Ai intersect, but the intersection of all Ai is empty.
1

4. Define

10

x=
i=1

Show that a) x <

1
1

10 3 1 + ( 10i3 )2

< y and b)

x+y
2

<

and

y=
i=0

1
1

.
10 3 1 + ( 10i3 )2

. (Hint: Relate these sums to an integral.)

5. Using the steps below, find the value of x2012 + x2012 , where x + x1 =

5+1
2 .

a) For any real r, show that |r + r1 | 2. What does this tell you about the given x?
b) Show that cos( 5 ) =

5+1
4 ,

3
e.g. compare sin( 2
5 ) and sin( 5 ).

c) Combine conclusions of parts a and b to express x and therefore the desired quantity in a suitable
form.
6. For n > 1, a configuration consists of 2n distinct points in a plane, n of them red, the remaining n
blue, with no three points collinear. A pairing consists of n line segments, each with one blue and one
red endpoint, such that each of the given 2n points is an endpoint of exactly one segment. Prove the
following.
a) For any configuration, there is a pairing in which no two of the n segments intersect. (Hint: consider
total length of segments.)
b) Given n red points (no three collinear), we can place n blue points such that any pairing in the
resulting configuration will have two segments that do not intersect. (Hint: First consider the case
n = 2.)
7. A sequence of integers cn starts with c0 = 0 and satisfies cn+2 = acn+1 + bcn for n 0, where a and
b are integers. For any positive integer k with gcd(k, b) = 1, show that cn is divisible by k for infinitely
many n.
8. Let f (x) be a polynomial with integer coefficients such that for each nonnegative integer n, f (n) = a
perfect power of a prime number, i.e., of the form pk , where p is prime and k a positive integer. (p and
k can vary with n.) Show that f must be a constant polynomial using the following steps or otherwise.
a) If such a polynomial f (x) exists, then there is a polynomial g(x) with integer coefficients such that
for each nonnegative integer n, g(n) = a perfect power of a fixed prime number.
b) Show that a polynomial g(x) as in part a must be constant.
9. Let N be the set of non-negative integers. Suppose f : N N is a function such that f (f (f (n))) <
f (n + 1) for every n N . Prove that f (n) = n for all n using the following steps or otherwise.
a) If f (n) = 0, then n = 0.
b) If f (x) < n, then x < n. (Start by considering n = 1.)
c) f (n) < f (n + 1) and n < f (n + 1) for all n.
d) f (n) = n for all n.

Solutions to the 2012 CMI BSc Entrance Examination


Part A: 5 problems 6 marks.

Part B: 7 out of 9 problems 10 marks.

A1. Find the number of real solutions to the equation x = 99 sin(x).


x
The number of solutions is the number of times the line y = 99
meets the graph of
y = sin(x). This can occur only for x [99, 99] because sin(x) has range [1, 1]. Also
sin(x) is periodic with period 2. For x 0, the two graphs meet twice in each cycle
of sin(x), both intersections occurring in the first half of the cycle. There are 50 such
half-cycles from x = 0 to x = 99, over intervals [0, 1], [2, 3], . . . , [98, 99]. So there are 100
non-negative solutions. Similarly there are 100 solutions 0 because both graphs are odd.
Since x = 0 is counted twice, the total number of solutions is 100 + 100 1 = 199.

A2. A differentiable function f : R R satisfies f (1) = 2, f (2) = 3 and f (3) = 1. Show


that f (x) = 0 for some x.
Since f is differentiable, it is continuous. By the intermediate value theorem, there is
a (2, 3) with f (a) = 2 = f (1). Now by Rolles theorem there is x (1, a) with f (x) = 0.
OR The continuous function f over the closed interval [1, 2] must attain its absolute
maximum, which cannot be at either endpoint (since f (2) > f (1) and f (2) > f (3)). So
the maximum must be at an interior point x and then f (x) = 0. OR By the mean
value theorem, f (y) = 1 > 0 for some y (1, 2) and f (z) = 2 < 0 for some z (2, 3).
So f (x) = 0 for some x (y, z) since for a differentiable f , the function f satisfies the
intermediate value property by Darbouxs theorem. (This is important to say because f
need not be continuous.)
A3. Show that

ln(12)
ln(18)

is irrational.

ln(12)
ln(18)

= log18 (12). Suppose this is rational, say = ab where a, b are integers with b = 0.
a
Then 18 b = 12, so 18a = 12b . By factoring into primes this gives 32a 2a = 3b 22b , which
by unique factorization can happen only if 2a = b and a = 2b. But this gives a = b = 0,
ln(2)
a contradiction. (Alternatively and similarly, prove that r = ln(3)
is irrational and show
that rationality of

ln(12)
ln(18)

ln 3+2 ln 2
2 ln 3+ln a

A4. Show that


lim

1+2r
2+r

would force r to be rational as well.)

x100 ln(x)
= 0.
ex tan1 ( 3 + sin x)

There is a positive constant c such that tan1 ( 3 +sin x) > c for any x, e.g. c = tan1 (0.04)
will work since > 3.12, sin(x) 1 and tan1 is an increasing function. Moreover
ln(x) < x for x > 0. So the given ratio is sandwiched between 0 and x101 /cex . Now use
LHospitals rule repeatedly.
1

A5. a) n identical chocolates are to be distributed among the k students in Tinkus class.
Find the probability that Tinku gets at least one chocolate, assuming that the n chocolates
are handed out one by one in n independent steps. At each step, one chocolate is given to
a randomly chosen student, with each student having equal chance to receive it.
P(Tinku gets at least one chocolate) = 1 P(Tinku gets none) = 1 (1 k1 )n , because in
each of the independent steps the probability of Tinku not getting a chocolate is 1 k1 .
b) Solve the same problem assuming instead that all distributions are equally likely. You
are given that the number of such distributions is n+k1
k1 . (Here all chocolates are considered interchangeable but students are considered different.)
There are (n1)+k1
distributions in which Tinku gets at least a chocolate: give Tinku a
k1
chocolate and then use the given formula to find number of distributions of the remaining
n
. OR
n 1 chocolates among k students. So the answer is (n1)+k1
/ n+k1
= n+k1
k1
k1
The number of distributions in which Tinku gets no chocolate = number of distributions
of n chocolates among the remaining k 1 students = n+k2
k2 . So the desired probability
n+k2
n+k1
n
is 1 k2 / k1 = n+k1 .

B1. a) Find a polynomial p(x) with real coefficients such that p( 2 + i) = 0.


Non-real
polynomial with
real coefficients occur in conjugate pairs. p(x) =
roots of a
2
(x ( 2 + i))(x ( 2 i)) = x 2 2x + 3 works.
b) Find a polynomial
q(x) with rational coefficients and having the smallest possible degree

such that
q(
2
+
i)
=
0. Show that any other polynomial with rational coefficients and

having 2 + i as a root has q(x) as a factor.

2 + i satisfies x2 2 2x + 3 = 0, i.e., x2 + 3 = 2 2x and so satisfies (x2 + 3)2 =


8x2 . So q(x) = (x2 + 3)2 8x2 works. A cubic with rational coefficients will not work
because, after
dividing by the necessarily rational leading coefficient, it must be of the
2
form (x 2 2x + 3)(x r). This forces the coefficients 3r and 2 2 r to be both
rational, which is impossible.

Let f (x) be a polynomial with rational coefficients such that f ( 2 + i) = 0. Divide f (x)
by q(x) using long division to get quotient a(x) andremainder b(x), both polynomials
with rational coefficients.
Using f ( 2 + i) = 0 and q( 2 + i) = 0 in the equation f (x) =
q(x)a(x) + b(x) gives b( 2 + i) = 0. Now if the remainder b(x)is a nonzero polynomial,
then it would have rational coefficients, degree less than 4 and 2 + i as a root. But we
just proved that this is impossible. Hence b(x) = 0, i.e., f (x) is a multiple of q(x).
B2. a) Let E, F, G and H respectively be the midpoints of the sides AB, BC, CD and
DA of a convex quadrilateral ABCD. Show that EFGH is a parallelogram whose area is
half that of ABCD.
2

Consider the diagonals AC and BD. By the basic proportionality theorem in triangle ABC,
we get that EF and AC are parallel and AC = 2 EF. Moreover, ABC and EBF are similar.
Using triangles ADC and HDG, we similarly get that AC is parallel to HG, AC = 2 HG.
Thus EF and HG are parallel. Likewise FG and EH are parallel (both parallel to BD), so
EFGH is a parallelogram. Also by similarity, Area(ABC) = 4 Area(EBF), Area(ADC) =
4 Area(HDG), Area(BAD) = 4 Area(EAH) and Area(BCD) = 4 Area(FCG). (Note. So
far convexity of ABCD is unnecessary. But the next steps need it, draw pictures and see.)
Area(EFGH) = Area(ABCD) [Area(EBF) + Area(FCG) + Area(GDH) + Area(HAE)]
= Area(ABCD) 41 [Area(ABC)+ Area(BCD) + Area(CDA) + Area(DAB)]
= Area(ABCD) 12 Area(ABCD) = 12 Area(ABCD).
b) Let E = (0, 0), F = (0, 1), G = (1, 1), H = (1, 0). Find all points A = (p, q) in the
first quadrant such that E, F, G and H respectively are the midpoints of the sides AB,
BC, CD and DA of a convex quadrilateral ABCD.
If A = (p, q) is such a point, then E = (0,0) being the midpoint of AB is equivalent to
having B = (p, q). Similarly we get C = (p, q 2), D = (2 p, q). In particular AC =
BD = 2, AC is vertical and BD horizontal. By the reasoning in part a), these facts imply
that the quadrilateral constructed from the midpoints of the sides of ABCD is a square of
side 1. So we just need to ensure that the listed coordinates make ABCD into a convex
quadrilateral. This happens if and only if p, q are both positive (which is given) and < 1.
It is easy to see that these conditions are sufficient to make ABCD a convex quadrilateral.
For necessity see the following (pictures will help). If p > 1 then A will be to the right of
H and so D to the left of H. If q > 1, then B will be below F and so C will be above F. If
p or q = 1, then three of the points A, B, C, D become collinear. In all cases ABCD will
not be a convex quadrilateral. If both p, q > 1, ABCD will even be self-intersecting.
B3. a) We want to choose subsets A1 , A2 , . . . , Ak of {1, 2, . . . , n} such that any two of the
chosen subsets have nonempty intersection. Show that the size k of any such collection of
subsets is at most 2n1 .
If a set A is in such a collection C, then the complement of A cannot be in C. Therefore
|C| 21 (total number of subsets of {1, 2, . . . , n}) = 21 2n = 2n1 .
b) For n > 2 show that we can always find a collection of 2n1 subsets A1 , A2 , . . . of
{1, 2, . . . , n} such that any two of the Ai intersect, but the intersection of all Ai is empty.
There are many ways to build such a collection, e.g., take all 2n1 subsets of {1, 2, . . . , n}
containing 1, remove the singleton set {1} and instead include its complement. OR
Note that for n = 3, the four sets {1, 2}, {2, 3}, {1, 3}, {1, 2, 3} give a (unique) solution.
For n > 3 take the union of each of these 4 sets with all 2n3 subsets of {4, . . . , n}. OR
For n = 2k + 1, take all subsets of {1, 2, . . . , n} of size > k. Any two of these will intersect.
n
Now use ni = ni
. For n = 2k, take all subsets of size > k along with half the subsets
of size k, namely those containing a fixed number. (Check the details.)
3

B4. Define
10

x=
i=1

Show that a) x <

1
1

10 3 1 + ( 10i3 )2
< y and b)

x+y
2

<

and

y=
i=0

1
1

.
10 3 1 + ( 10i3 )2

. (Hint: Relate these sums to an integral.)

a) Let f (t) = 1/(1 + t2 ). Then y and x are respectively the left and right hand
Riemann
1
sums for f over the interval [0, 3 ] using 10 equal parts, each of width 1/10 3. Since f is
a positive decreasing function, y overestimates the area under
f over the given interval
and

1/ 3

1/ 3

x underestimates it. The area under f over [0, 13 ] is 0


f (t)dt = tan1 (t)|0
= 6 , so
x < 6 < y. Note. Different normalizations are possible for f , e.g., the more simpleminded
choice f (t) = 1013 1+( 1t )2 considered over the interval [0,10] will work too.
10

b) x+y
can be interpreted as the sum of areas of 10 trapezoids as follows. Dividing
2
[0, 13 ] into 10 equal parts, let the i-th subinterval be [ti1 , ti ] with i = 0, 1, . . . , 10. Then
the i-th trapezoid has base [ti1 , ti ] and it has two vertical sides, the left one of height
f (ti1 ) and the right one of height f (ti ) (draw a picture and see). So we have to prove
that the total area of trapezoids is less than the area under f . For this we should check
concavity of f (draw pictures and see why). Check that over the interval (0, 13 ), we have
2

6t 2
f (t) = (1+t
2 )3 < 0, so f is concave down and hence each trapezoid lies completely below
the graph of f .

B5. Using the steps below, find the value of x2012 + x2012 , where x + x1 =

5+1
2 .

a) For any real r, show that |r + r1 | 2. What does this tell you about the given x?
Because of the absolute value we may assume that r > 0 by replacing r with r if
necessary.

2
1
Now use AM-GM inequality or the fact that ( r 1/r) 0. Since x+x = 5+1
< 2,
2
given x must be a non-real (complex) number.
b) Show that cos( 5 ) =

5+1
4 ,

3
e.g. compare sin( 2
5 ) and sin( 5 ).

Let = 5 . Then sin(2) = sin( 2) = sin(3). Using the formulas for sin(2) and
sin(3), canceling sin (it is nonzero) and substituting sin2 = 1 cos2, gives the
quadratic equation 4 cos2 2 cos 1 = 0. Since cos > 0, we get cos = 5+1
4 .
c) Combine conclusions of parts a and b to express x and therefore the desired quantity
in a suitable form.
Let x = dei = d(cos + isin ). Then x1 = d1 ei = d1 (cos i sin ). Adding
i
and using that x + x1 = 5+1
= 2 cos( 5 ), we get d = 1 and = . So x = e 5 and
2

2
2
51
2
x2012 + x2012 = 2 cos( 2012
)
=
2
cos(402
+
)
=
2
cos(
)
=
2
cos
(
)

1
=
5
5
5
5
2 .
4

B6. For n > 1, a configuration consists of 2n distinct points in a plane, n of them red, the
remaining n blue, with no three points collinear. A pairing consists of n line segments, each
with one blue and one red endpoint, such that each of the given 2n points is an endpoint
of exactly one segment. Prove the following.
a) For any configuration, there is a pairing in which no two of the n segments intersect.
(Hint: consider total length of segments.)
For any configuration, there are only finitely many pairings. Choose one with least possible
total length of segments. Here no two of the n segments can interest, because if RB and
R B intersect in point X then we get a contradiction as follows. Using triangle inequality
in triangles RXB and R XB, we get RB + R B < RB + R B (draw a picture). So
replacing RB and R B with R B and RB would give a pairing with smaller total length.
b) Given n red points (no three collinear), we can place n blue points such that any pairing
in the resulting configuration will have two segments that do not intersect. (Hint: First
consider the case n = 2.)
For n = 2, place the two blue points on opposite sides of the line passing through the given
two red points. There are two possible pairings and the two segments in either one do not
intersect. We use a similar idea in general. Given n red points, find a triangle ABC such
that A is a red point and all other red points are inside triangle ABC. (This is always
possible. Why?) Place one blue point at B and all other blue points in the region opposite
to triangle ABC at vertex C. (More precisely, let C be between A and A and also between
B and B . Place the remaining blue points inside triangle A CB .) Now in any pairing, if
A and B are connected, then AB will not intersect any other segment. Otherwise the two
segments having A and B as vertices will not intersect. Draw a picture to see this.
B7. A sequence of integers cn starts with c0 = 0 and satisfies cn+2 = acn+1 + bcn for
n 0, where a and b are integers. For any positive integer k with gcd(k, b) = 1, show that
cn is divisible by k for infinitely many n.
Consider pairs of consecutive entries of the sequence modulo k, i.e., (
cn , cn+1 ), where a

2
denotes a modulo k. Since there are only finitely many possibilities (namely k ), some pair
of consecutive residues will repeat. Suppose (
ci , ci+1 ) = (
ci+p , ci+p+1 ) for some i. We will
show that in fact the previous equation holds for all i, i.e., whole sequence of consecutive
pairs is periodic. This will prove in particular that (
c0 , c1 ) = (
cp , cp+1 ) = (
c2p , c2p+1 ) = .
Since c0 = 0 is divisible by k, so is cip for all i.
The equation cn+2 = acn+1 + bcn shows that b
cn = cn+2 a
cn+1 . Now gcd(k, b) = 1 means

b is invertible modulo k, i.e., there is a b with bb = 1. Therefore cn = b (


cn+2 a
cn+1 ).
Thus knowing a pair of consecutive residues uniquely determines the previous residue
(this is why we considered pairs of residues). Therefore (
ci , ci+1 ) = (
ci+p , ci+p+1 ) implies
(
ci1 , ci ) = (
ci+p1 , ci+p ) and (by the given recurrence) (
ci+1 , ci+2 ) = (
ci+p+1 , ci+p+2 ).
Thus the whole sequence (
cn , cn+1 ) becomes periodic as soon as a single such pair repeats.
5

B8. Let f (x) be a polynomial with integer coefficients such that for each nonnegative
integer n, f (n) = a perfect power of a prime number, i.e., of the form pk , where p is prime
and k a positive integer. (p and k can vary with n.) Show that f must be a constant
polynomial using the following steps or otherwise.
a) If such a polynomial f (x) exists, then there is a polynomial g(x) with integer coefficients
such that for each nonnegative integer n, g(n) = a perfect power of a fixed prime number.
Write f (x) = an xn + an1 xn1 + + a1 x + a0 . Then a0 = f (0) = pk for some prime
p and integer k > 0. Define g(x) = f (px). Then g(x) is a polynomial such that for each
nonnegative integer n, g(n) = f (pn) = a perfect power of a prime number. This prime
number has to be p, because by evaluating we see that g(n) = f (pn) is divisible by p.
b) Show that a polynomial g(x) as in part a must be constant.
Let g(x) = bn xn + bn1 xn1 + + b1 x + b0 . Then b0 = g(0) = pk . Consider g(mpk+1 ) =
bn (mpk+1 )n +bn1 (mpk+1 )n1 + +b1 (mpk+1 )+pk . Clearly for each non-negative integer
m, this expression is divisible by pk , but not by pk+1 (since it is pk modulo pk+1 ). This
forces g(mpk+1 ) = pk for all m, since it must be a perfect power of p. Thus the polynomial
g takes the value pk infinitely often, so it must be identically equal to pk . (Otherwise the
polynomial g(x) pk would have infinitely many roots.) To finish the problem, note that
since g(x) = f (px) is constant, f (x) must be constant by the same logic.
B9. Let N be the set of non-negative integers. Suppose f : N N is a function such that
f (f (f (n))) < f (n + 1) for every n N . Prove that f (n) = n for all n using the following
steps or otherwise.
a) If f (n) = 0, then n = 0.
Let f (n) = 0. If n > 0, then n 1 is in the domain of f and f (f (f (n 1))) < f (n) = 0,
which is a contradiction, since 0 is the smallest possible value of f . (Note that this does
NOT prove that f (0) = 0, only that if f (some n) = 0, then that n = 0. In fact proving
f (0) = 0 along with part a would essentially solve the problem, see below.)
b) If f (x) < n, then x < n. (Start by considering n = 1.)
Induction on n. If n = 1, then this is just part a. Assuming the statement up to n we need
to prove that if f (x) < n + 1, then x < n + 1. If f (x) < n, then by induction x < n, so
x < n + 1. So let f (x) = n. If x = 0, we are done. Otherwise f (f (f (x 1))) < f (x) = n
and by using induction thrice we get in succession f (f (x 1)) < n, then f (x 1) < n and
then x 1 < n, i.e., x < n + 1 as desired.
c) f (n) < f (n + 1) and n < f (n + 1) for all n.
Apply part b to f (f (f (m))) < f (m + 1) (with x = f (f (m)) and n = f (m + 1)) to get
6

f (f (m)) < f (m + 1). Apply part b to this with x = f (m) and n = f (m + 1) to get
f (m) < f (m + 1). Again apply part b to get m < f (m + 1).
d) f (n) = n for all n.
By part c, f is increasing and f (n) n. If f (n) > n, then f (f (n)) > f (n) (since f
is increasing) and so f (f (n)) > n, i.e., f (f (n)) n + 1. Again, since f is increasing,
f (f (f (n))) f (n + 1), a contradiction.
Alternative solution after part a. Let us prove f (0) = 0. We know that f (n) = 0
implies n = 0, so n > 0 implies f (n) > 0. Applying this to any positive f (k), we get
f (f (k)) > 0. Denoting f (f (k)) = x, we therefore get f (f (f (x 1))) < f (x) = f (f (f (k))).
This means that for k such that f (f (f (k))) is the smallest number in {f (f (f (n)))|n 0},
we must have f (k) = 0. In particular 0 is in the range of f , so by part a f (0) = 0.
Since f (n) = 0 for no other n, we may restrict the function f by deleting 0 from the
domain and the range. The resulting function would satisfy f (f (f (n))) < f (n + 1) for
every n > 0. Repeat the reasoning substituting 1 (the new lowest element of the domain
and the range) for 0 and conclude f (1) = 1. Then restrict to n > 1 and show f (2) = 2
and so on.

2013 Entrance Examination for BSc Programmes at CMI

Part A. (10 problems 5 points = 50 points.) Attempt all questions in this part
before going to part B. Carefully read the details of marking scheme given
below. Note that wrong answers will get negative marks!
In each problem you have to fill in 4 blanks as directed. Points will be given based only on
the filled answer, so you need not explain your answer. Each correct answer gets 1 point
and having all 4 answers correct will get 1 extra point for a total of 5 points per problem.
But each wrong/illegible/unclear answer will get minus 1 point. Negative points from any
problem will be counted in your total score, so it is better not to guess! If you are unsure
about a part, you may leave it blank without any penalty. If you write something and then
want it not to count, cross it out and clearly write no attempt next to the relevant part.
1. For sets A and B, let f : A B and g : B A be functions such that f (g(x)) = x for
each x. For each statement below, write whether it is TRUE or FALSE.
a) The function f must be one-to-one.

Answer:

b) The function f must be onto.

Answer:

c) The function g must be one-to-one.

Answer:

d) The function g must be onto.

Answer:

2. Let f : R R be a function, where R is the set of real numbers. For each statement
below, write whether it is TRUE or FALSE.

a) If |f (x) f (y)| 39|x y| for all x, y then f must be continuous everywhere.

Answer:

b) If |f (x) f (y)| 39|x y| for all x, y then f must be differentiable everywhere.

Answer:

c) If |f (x) f (y)| 39|x y|2 for all x, y then f must be differentiable everywhere.

Answer:

d) If |f (x) f (y)| 39|x y|2 for all x, y then f must be constant.

Answer:

3. Let S be a circle with center O. Suppose A, B are points on the circumference of S with
AOB = 120 . For triangle AOB, let C be its circumcenter and D its orthocenter (i.e.,
the point of intersection of the three lines containing the altitudes). For each statement
below, write whether it is TRUE or FALSE.

a) The triangle AOC is equilateral.

Answer:

b) The triangle ABD is equilateral.

Answer:

c) The point C lies on the circle S.

Answer:

d) The point D lies on the circle S.

Answer:

4. A polynomial f (x) with real coefficients is said to be a sum of squares if we can write
f (x) = p1 (x)2 + + pk (x)2 , where p1 (x), . . . , pk (x) are polynomials with real coefficients.
For each statement below, write whether it is TRUE or FALSE.

a) If a polynomial f (x) is a sum of squares, then the coefficient of every odd power of x in
f (x) must be 0.

Answer:

b) If f (x) = x2 + px + q has a non-real root, then f (x) is a sum of squares.

Answer:

c) If f (x) = x3 + px2 + qx + r has a non-real root, then f (x) is a sum of squares.

Answer:

d) If a polynomial f (x) > 0 for all real values of x, then f (x) is a sum of squares.

Answer:

5. There are 8 boys and 7 girls in a group. For each of the tasks specified below, write an
expression for the number of ways of doing it. Do NOT try to simplify your answers.

a) Sitting in a row so that all boys sit contiguously and all girls sit contiguously, i.e., no
girl sits between any two boys and no boy sits between any two girls

Answer:

b) Sitting in a row so that between any two boys there is a girl and between any two girls
there is a boy

Answer:

c) Choosing a team of six people from the group

Answer:

d) Choosing a team of six people consisting of unequal number of boys and girls

Answer:

6. Calculate the following integrals whenever possible. If a given integral does not exist,
state so. Note that [x] denotes the integer part of x, i.e., the unique integer n such that
n x < n + 1.

a)

4
1

x2 dx

Answer:

b)

3
[x]2 dx
1

Answer:

c)

2 2
[x ]dx
1

Answer:

d)

1 1
dx
1 x2

Answer:

7. Let A, B, C be angles such that eiA , eiB , eiC form an equilateral triangle in the complex
plane. Find values of the given expressions.

a) eiA + eiB + eiC

Answer:

b) cos A + cos B + cos C

Answer:

c) cos 2A + cos 2B + cos 2C

Answer:

d) cos2 A + cos2 B + cos2 C

Answer:

8. Consider the quadratic equation x2 + bx + c = 0, where b and c are chosen randomly


from the interval [0,1] with the probability uniformly distributed over all pairs (b, c). Let
p(b) = the probability that the given equation has a real solution for given (fixed) value of
b. Answer the following questions by filling in the blanks.

a) The equation x2 + bx + c = 0 has a real solution if and only if b2 4c is

Answer:

b) The value of p( 12 ), i.e., the probability that x2 +

x
2

+ c = 0 has a real solution is

Answer:

c) As a function of b, is p(b) increasing, decreasing or constant?

Answer:

d) As b and c both vary, what is the probability that x2 + bx + c = 0 has a real solution?

Answer:

9. Let R = the set of real numbers. A continuous function f : R R satisfies f (1) = 1,


f (2) = 4, f (3) = 9 and f (4) = 16. Answer the independent questions below by choosing
the correct option from the given ones.

a) Which of the following values must be in the range of f ?


Options: 5
25
both
neither

Answer:

b) Suppose f is differentiable. Then which of the follwing intervals must contain an x such
that f (x) = 2x ?
Options: (1,2)
(2,4)
both
neither

Answer:

c) Suppose f is twice differentiable. Which of the following intervals must contain an x


such that f (x) = 2 ?
Options: (1,2)
(2,4)
both
neither

Answer:

d) Suppose f is a polynomial, then which of the following are possible values of its degree?
Options: 3
4
both
neither

Answer:

10. Let
f (x) =

x4
(x 1)(x 2) (x n)

where the denominator is a product of n factors, n being a positive integer. It is also given
that the X-axis is a horizontal asymptote for the graph of f . Answer the independent
questions below by choosing the correct option from the given ones.

a) How many vertical asymptotes does the graph of f have?


Options: n
less than n
more than n
impossible to decide

Answer:

b) What can you deduce about the value of n ?


Options: n < 4
n=4
n>4
impossible to decide

Answer:

c) As one travels along the graph of f from left to right, at which of the following points
is the sign of f (x) guaranteed to change from positive to negative?
Options: x = 0
x=1
x=n1
x=n

Answer:

d) How many inflection points does the graph of f have in the region x < 0 ?
Options: none
1
more than 1
impossible to decide
(Hint: Sketching is better than calculating.)

Answer:

10

Part B. (Problems 14 15 points + problems 56 20 points = 100 points.) Solve


these problems in the space provided for each problem after this page. You may solve only
part of a problem and get partial credit. Clearly explain your entire reasoning. No
credit will be given without reasoning.
1. In triangle ABC, the bisector of angle A meets side BC in point D and the bisector of
angle B meets side AC in point E. Given that DE is parallel to AB, show that AE = BD
and that the triangle ABC is isosceles.
2. A curve C has the property that the slope of the tangent at any given point (x, y) on
2
+y 2
.
C is x 2xy
a) Find the general equation for such a curve. Possible hint: let z = xy .
b) Specify all possible shapes of the curves in this family. (For example, does the family
include an ellipse?)
3. A positive integer N has its first, third and fifth digits equal and its second, fourth and
sixth digits equal. In other words, when written in the usual decimal system it has the
form xyxyxy, where x and y are the digits. Show that N cannot be a perfect power, i.e.,
N cannot equal ab , where a and b are positive integers with b > 1.
4. Suppose f (x) is a function from R to R such that f (f (x)) = f (x)2013 . Show that there
are infinitely many such functions, of which exactly four are polynomials. (Here R = the
set of real numbers.)
1
5. Consider the function f (x) = ax + x+1
, where a is a positive constant. Let L = the
1
largest value of f (x) and S = the smallest value of f (x) for x [0, 1]. Show that LS > 12
for any a > 0.

6. Define fk (n) to be the sum of all possible products of k distinct integers chosen from
the set {1, 2, . . . , n}, i.e.,
fk (n) =

i1 i2 . . . ik .
1i1 <i2 <...<ik n

a) For k > 1, write a recursive formula for the function fk , i.e., a formula for fk (n) in
terms of f (m), where < k or ( = k and m < n).
b) Show that fk (n), as a function of n, is a polynomial of degree 2k.
c) Express f2 (n) as a polynomial in variable n.

11

Solutions for 2013 Entrance Examination for BSc Programmes at CMI

Part A. (10 problems 5 points = 50 points.) Attempt all questions in this part
before going to part B. Carefully read the details of marking scheme given
below. Note that wrong answers will get negative marks!
In each problem you have to fill in 4 blanks as directed. Points will be given based only on
the filled answer, so you need not explain your answer. Each correct answer gets 1 point
and having all 4 answers correct will get 1 extra point for a total of 5 points per problem.
But each wrong/illegible/unclear answer will get minus 1 point. Negative points from any
problem will be counted in your total score, so it is better not to guess! If you are unsure
about a part, you may leave it blank without any penalty. If you write something and then
want it not to count, cross it out and clearly write no attempt next to the relevant part.
1. For sets A and B, let f : A B and g : B A be functions such that f (g(x)) = x for
each x. For each statement below, write whether it is TRUE or FALSE.
a) The function f must be one-to-one.
b) The function f must be onto.
c) The function g must be one-to-one.
d) The function g must be onto.
Answer: FTTF.
If g(x1 ) = g(x2 ), then x1 = f (g(x1 )) = f (g(x2 )) = x2 , so g is one-to-one. Also f is onto
because each x B is in the image of f , namely x = f (g(x)). The other two statements
are false, e.g. by constructing an example in which A is a larger finite set than B.
2. Let f : R R be a function, where R is the set of real numbers. For each statement
below, write whether it is TRUE or FALSE.
a) If |f (x) f (y)| 39|x y| for all x, y then f must be continuous everywhere.
b) If |f (x) f (y)| 39|x y| for all x, y then f must be differentiable everywhere.
c) If |f (x) f (y)| 39|x y|2 for all x, y then f must be differentiable everywhere.
d) If |f (x) f (y)| 39|x y|2 for all x, y then f must be constant.
Answer: TFTT
In parts a and b, we have |f (x) f (a)| sandwiched between 39|x a|. As x a,
39|x a| 0 and hence f (x) f (a) 0, so f is continuous. But it need not be
differentiable, e.g. f (x) = |x| satisfies f (x) f (y) = |x| |y| |x y| 39|x y|. But f
is not differentiable at 0.
(a)
In parts c and d, we have | f (x)f
| 39|x a|, so by reasoning as for part a, we have
xa

limxa

f (x)f (a)
xa

= 0, i.e., f (a) = 0 for all a, so f is a constant function.


1

3. Let S be a circle with center O. Suppose A, B are points on the circumference of S with
AOB = 120 . For triangle AOB, let C be its circumcenter and D its orthocenter (i.e.,
the point of intersection of the three lines containing the altitudes). For each statement
below, write whether it is TRUE or FALSE.
a) The triangle AOC is equilateral.
b) The triangle ABD is equilateral.
c) The point C lies on the circle S.
d) The point D lies on the circle S.
Answer: TTTT
Draw a picture and see that the bisector of AOB splits this angle into two angles of
60 each and meets the circle, say in point C . Now the triangles OAC and OBC are
both equilateral, so AC = OC = BC , making C = C, the cirumcenter of triangle
AOB. Similarly, letting CD be a diameter of the circle S, it is easy to deduce that
AOD = BOD = 120 and that triangle ABD is also equilateral with O as its
centroid. Hence CD AB, line BO AD and line AO BD , making D = D, the
orthocenter of triangle AOB.
4. A polynomial f (x) with real coefficients is said to be a sum of squares if we can write
f (x) = p1 (x)2 + + pk (x)2 , where p1 (x), . . . , pk (x) are polynomials with real coefficients.
For each statement below, write whether it is TRUE or FALSE.
a) If a polynomial f (x) is a sum of squares, then the coefficient of every odd power of x in
f (x) must be 0.
b) If f (x) = x2 + px + q has a non-real root, then f (x) is a sum of squares.
c) If f (x) = x3 + px2 + qx + r has a non-real root, then f (x) is a sum of squares.
d) If a polynomial f (x) > 0 for all real values of x, then f (x) is a sum of squares.
Answer: FTFT
2

For part b, complete the square to get f (x) = x2 + px + q = (x + p2 )2 + ( 4qp


4 ), which is a
sum of squares since 4q p2 > 0 due to the roots being non-real. Since p need not be 0, this
disproves part a. For part d, since all roots of f are non-real and occur in conjugate pairs,
f (x) = a product of quadratic polynomials each of which is a sum of squares by part b.
For part c, note that f (x) as x , so in particular f (x) takes negative values
and hence can never be a sum of squares. (This applies to any odd degree polynomial.)
5. There are 8 boys and 7 girls in a group. For each of the tasks specified below, write an
expression for the number of ways of doing it. Do NOT try to simplify your answers.
a) Sitting in a row so that all boys sit contiguously and all girls sit contiguously, i.e., no
girl sits between any two boys and no boy sits between any two girls.
Answer: 2 8! 7! (The factor of 2 arises because the two blocks of boys and girls can
switch positions.)
2

b) Sitting in a row so that between any two boys there is a girl and between any two girls
there is a boy
Answer: 8! 7! (There is no factor of 2 because there must be a boy at each end.)
c) Choosing a team of six people from the group

Answer:

15
6

d) Choosing a team of six people consisting of unequal number of boys and girls
Answer:

15
6

8
3

7
3

8
6

8
5

7
1

8
4

7
2

8
2

7
4

8
1

7
5

7
6

6. Calculate the following integrals whenever possible. If a given integral does not exist,
state so. Note that [x] denotes the integer part of x, i.e., the unique integer n such that
n x < n + 1.
a)

4
1

x2 dx =

x3 4
3 |1

= 21 using the fundamental theorem of calculus.

3
[x]2 dx
1

= 1(12 ) + 1(22 ) = 5 = area under the piecewise constant function [x]2

2
c) 1 [x2 ]dx = 1( 2 1) + 2( 3 2) + 3(2 3) = 5 2 3 since the function [x]2

is constant on intervals [1, 2), [ 2, 3), [ 3, 2), taking values 1, 2, 3 respectively.


b)

d) 1 x12 dx = 2 limt0+ t x12 dx = 2 limt0+ (1 + 1t ) = . The fundamental theorem


does not apply over the interval [1, 1] because x12 goes to in the interval. It is also ok
to answer that the integral does not exist (as a real number).

7. Let A, B, C be angles such that eiA , eiB , eiC form an equilateral triangle in the complex
plane. Find values of the given expressions.
a) eiA + eiB + eiC = 0 by taking the vector sum of the three points on the unit circle.
b) cos A + cos B + cos C = 0 = real part of eiA + eiB + eiC , which is 0 by part a.
c) cos 2A+cos 2B+cos 2C = 0 because the points e2iA , e2iB , e2iC on the unit circle also form
an equilateral triangle in the complex plane, since taking B = A + (2/3), C = A + (4/3),
we get 2B = 2A + (4/3) and 2C = 2A + (8/3) = 2A + (2/3) + 2 and the last term 2
does not change the position of the point.
d) cos2 A + cos2 B + cos2 C = 32 because, using the formula for cos 2 in part c, we get
cos2 A + cos2 B + cos2 C = sin2 A + sin2 B + sin2 C and the sum of the LHS and the RHS
in this equation is 3.
8. Consider the quadratic equation x2 + bx + c = 0, where b and c are chosen randomly
from the interval [0,1] with the probability uniformly distributed over all pairs (b, c). Let
p(b) = the probability that the given equation has a real solution for given (fixed) value of
b. Answer the following questions by filling in the blanks.
3

a) The equation x2 + bx + c = 0 has a real solution if and only if b2 4c is 0.


b) The value of p( 21 ), i.e., the probability that x2 +
Answer:
which is

x
2

+ c = 0 has a real solution is

1
1
2
16 since a real solution occurs precisely when b 4c = 4 4c
1 th
fraction of the interval [0, 1] over which c ranges.
16

0, i.e., 0 c

1
16 ,

c) As a function of b, is p(b) increasing, decreasing or constant?


Answer: increasing, because b2 4c 0 if and only if 0 c
increasing for 0 b 1.

b2
4 ,

so p(b) =

b2
4 ,

which is

d) As b and c both vary, what is the probability that x2 + bx + c = 0 has a real solution?
Answer: This is the fraction of the area of the unit square [0, 1] [0, 1] that is occupied by
2
the region b2 4c 0, i.e., it is the area under the parabola c = b4 from b = 0 to b = 1,
1 2
1
.
which is 0 b4 db = 12
9. Let R = the set of real numbers. A continuous function f : R R satisfies f (1) = 1,
f (2) = 4, f (3) = 9 and f (4) = 16. Answer the independent questions below by choosing
the correct option from the given ones.
a) Which of the following values must be in the range of f ?
Options: 5
25
both
neither
Answer: 5, by the intermediate value theorem, e.g., over the interval [2,3]. Also f (x) need
not take the value 25, e.g., take f (x) = x2 for x < 4 and f (x) = 16 for x 4.
b) Suppose f is differentiable. Then which of the follwing intervals must contain an x such
that f (x) = 2x ?
Options: (1,2)
(2,4)
both
neither
Answer: both
c) Suppose f is twice differentiable. Which of the following intervals must contain an x
such that f (x) = 2 ?
Options: (1,2)
(2,4)
both
neither
Answer: (2,4)
d) Suppose f is a polynomial, then which of the following are possible values of its degree?
Options: 3
4
both
neither
Answer: 4
For parts b,c and d, let g(x) = f (x) x2 . We have g(1) = g(2) = g(3) = g(4) = 0. For
part b, applying Rolles theorem to g(x) gives g (x) = 0 for some values of x in each of the
intervals (1, 2), (2, 3), (3, 4). For these values of x, we have f (x) = g (x) + 2x = 2x.
Far part c, take from part b values r (2, 3) and s (3, 4) with g (r) = 0 = g (s). Applying
Rolles theorem to g (x) in the interval [r, s], we get for some x (r, s) (2, 4) the equality
g (x) = 0 and so f (x) = g (x) + 2 = 2. There need not be an x (1, 2) with f (x) = 2,
i.e., g (x) = 0. There are many ways to arrange this, for example let g(x) = sin(x). Then
4

g (x) = 2 sin(x), which is 0 only when x is an integer, in particular g (x) = 0 for any
x (1, 2).
For part d, note that g(x), now being a polynomial vanishing at 1, 2, 3 and 4, must be
divisible by (x 1)(x 2)(x 3)(x 4). So g(x), if non-zero, must have degree at least 4.
Thus f (x) = x2 or a polynomial of degree at least 4.
10. Let
f (x) =

x4
(x 1)(x 2) (x n)

where the denominator is a product of n factors, n being a positive integer. It is also given
that the X-axis is a horizontal asymptote for the graph of f . Answer the independent
questions below by choosing the correct option from the given ones.
a) How many vertical asymptotes does the graph of f have?
Options: n
less than n
more than n
impossible to decide
Answer: n, at x = 1, 2, . . . , n.
b) What can you deduce about the value of n ?
Options: n < 4
n=4
n>4
impossible to decide
Answer: n > 4, because limx f (x) = 0 and for this to happen, the degree of the
denominator of f (x) must be greater than that of the numerator.
c) As one travels along the graph of f from left to right, at which of the following points
is the sign of f (x) guaranteed to change from positive to negative?
Options: x = 0
x=1
x=n1
x=n
Answer: x = n1, because f (x) is positive for x > n and f (x) changes sign precisely when
it passes through x = 1, 2 . . . , n. Note that the sign of f (x) for x < 0 and for x (0, 1)
depends on the parity of n.
d) How many inflection points does the graph of f have in the region x < 0 ?
Options: none
1
more than 1
impossible to decide
(Hint: Sketching is better than calculating.)
Answer: more than 1. Note that f (x) = 0 only at x = 0, with multiplicity 4. Without loss
of generality, let n be even. (If n is odd, the reasoning is completely parallel, see note at
the end.) Now f (x) > 0 for x < 1 except at x = 0 and f has all derivatives for x < 1. Due
to the multiple root at x = 0, the graph of f must be concave up (i.e. f (x) > 0) near
x = 0. Further, as x , the values of f (x) stay positive and 0. Therefore, as one
traces the graph leftward from the origin, it must become concave down at least once and
eventually concave up again so as to approach the X-axis from above. (Note: If n is odd,
f (x) < 0 for x < 1 except at x = 0. As one traces the graph leftward from the origin, the
function is initially as well as eventually concave down and must be concave up at least
once in-between so as to approach the X-axis from below.)
5

Part B. (Problems 14 15 points + problems 56 20 points = 100 points.) Solve


these problems in the space provided for each problem after this page. You may solve only
part of a problem and get partial credit. Clearly explain your entire reasoning. No
credit will be given without reasoning.
1. In triangle ABC, the bisector of angle A meets side BC in point D and the bisector of
angle B meets side AC in point E. Given that DE is parallel to AB, show that AE = BD
and that the triangle ABC is isosceles.
Answer: EAD = DAB = EDA, the first equality because AD bisects EAB and
the second because alternate angles made by line AD intersecting parallel lines DE and
AB are equal. Thus EAD is isosceles with EA = ED. Similarly ED = DB using the
fact that BE bisects DBA also intersects parallel lines DE and AB. Therefore EA =
CD
ED = DB. Now by the basic proportionality theorem, CE
EA = DB . As the denominators
EA and DB are equal, the numerators must be equal as well, i.e., CE = CD. Finally,
CA = CE + EA = CD + DB = CB, so ABC is isosceles.
2. A curve C has the property that the slope of the tangent at any given point (x, y) on
2
+y 2
C is x 2xy
.
a) Find the general equation for such a curve. Possible hint: let z = xy .
b) Specify all possible shapes of the curves in this family. (For example, does the family
include an ellipse?)
Answer: The defining property of the curve C is equivalent to the differential equation
dy
x2 +y 2
=
= 12 ( xy + xy ). It is convenient to let z = y/x, so the equation becomes
dx
2xy
dy
dx

= 21 ( z1 + z). To get this in terms of only x and z, differentiate z = y/x with respect to x
2
dy
dy
dz
= x1 dx
xy2 = x1 ( dx
z) = x1 ( 12 ( z1 + z) z) = x1 1z
to get dx
2z , where we have substituted
dy
for dx
using the differential equation and then simplified. Separating the variables and
2zdz
2
integrating, we get dx
x =
1z 2 , which gives log |x| = log |1 z |+ a constant, i.e.,
K

log |1 z 2 | = log |x| + K = log |x|1 + K. Exponentiating, we get 1 z 2 = ex =


2

c
x,

where c is a nonzero constant. Substituting z = y/x, we get 1 xy 2 = xc , i.e., x2 y 2 = cx.


To be precise, we have to delete the points (0, 0) and (c, 0) from this solution, because
2
dy
+y 2
for the given equation dx
= x 2xy
to make sense, both x and y must be nonzero. If the
dy
equation were given as 2xy dx
= x2 + y 2 , then this issue would not arise.
2

To see the shape of the curve, complete the square to get (x 2c )2 y 2 = c4 , which is
a hyperbola when c = 0. (Note: By differentiating x2 y 2 = cx, it is easy to see that
2
dy
+y 2
2xc
= x 2xy
and that this holds even when c = 0. Thus we get the two straight lines
dx =
y
y = x also as solutions. The reason the above answer missed this possibility was because
we put 1 z 2 in the denominator while separating variables, which precludes z = 1, i.e.,
y = x. To be precise, even here we have to delete the origin from the two lines.)
6

3. A positive integer N has its first, third and fifth digits equal and its second, fourth and
sixth digits equal. In other words, when written in the usual decimal system it has the
form xyxyxy, where x and y are the digits. Show that N cannot be a perfect power, i.e.,
N cannot equal ab , where a and b are positive integers with b > 1.
Answer: We have N = (105 + 103 + 10)x + (104 + 102 + 1)y = 10101(10x + y) =
3 7 13 37 (10x + y). Therefore for N to be a perfect power, the primes 3,7,13,37
must all occur (and in fact with equal power) as factors in the prime factorization of
10x + y. In particular 10x + y 10101. But since x and y are digits, each is between 0
and 9, so 10x + y 99. So N cannot be a perfect power.
4. Suppose f (x) is a function from R to R such that f (f (x)) = f (x)2013 . Show that there
are infinitely many such functions, of which exactly four are polynomials. (Here R = the
set of real numbers.)
Answer: If f is a polynomial, then we make two cases. (i) If f (x) = a constant c, then
the given condition is equivalent to c = c2013 , which happens precisely for three values
of c, namely c = 0, 1, 1 (since we have c(c2012 1) = 0, so c = 0 or c2012 = 1). Thus
there are three constant functions with the given property. (ii) If f (x) is a non-constant
polynomial, then consider its range set A = {f (x)|x R}. Now for all a A, we have by
the given property f (a) = a2013 . So the polynomial f (x) x2013 has all elements of A as
its roots. Since there are infinitely many values in A (e.g. applying the intermediate value
theorem because f is continuous), the polynomial f (x) x2013 has infinitely many roots
and thus must be the zero polynomial, i.e., f (x) = x2013 for all real number x.
Note: One can also deduce that the degree of f must be 0 or 2013 by equating the degrees
of f (f (x)) and f (x)2013 . Then, in the non-constant case, it is possible to argue first that
the leading coefficient is 1 and then that all other coefficients must be 0.
To find infinitely many function with the given property, define f (0) = 0, f (1) = 1 and
f (1) = 1. For every other real number x, arbitrarily define f (x) to be 0, 1 or 1. It is
easy to see that any such function satisfies the given property. (Other answers are possible,
e.g., more systematically, observe that f (a) = a2013 for at least one real number a (e.g.,
i
any number in the range of f ) and then this forces f (x) = x2013 for all x S = {a2013 |i =
0, 1, 2, . . .}. We use this as follows. Fix a real number a. Then define f (x) = x2013 for all
i
x S = {a2013 |i = 0, 1, 2, . . .}. For all x S, simply define f (x) = any element of the set
S, e.g., a itself will do.)
1
, where a is a positive constant. Let L = the
5. Consider the function f (x) = ax + x+1
1
largest value of f (x) and S = the smallest value of f (x) for x [0, 1]. Show that LS > 12
for any a > 0.
1
Answer: Let f (x) = ax + x+1
. We wish to understand the minimum and maximum of
1
this function in the interval [0, 1]. Now f (0) = 1, f (1) = a + 21 and f (x) = a (x+1)
2.
Over the interval [0, 1], the value of f (x) increases from a 1 at x = 0 to a 14 at x = 1.

We should consider what happens to the sign of f (x). For this we consider the following
cases.
(1) Suppose a 1/4. Because 1/(x + 1)2 1/4 on the interval [0, 1], f (x) 0, so the
maximum is at 0 and the minimum is at x = 1. So the difference is 1 (1/2 + a) =
1/2 a 1/4 1/12.
(2) Suppose a 1. Then f (x) 0 on the interval [0, 1], so maximum is at 1 and minimum
at 0. We get a + 1/2 1 = a 1/2 1/2 1/12.
(3) Suppose 1/4 a 1. Now f (x) = 0 at x
= 1a 1. For this range of a, x
[0, 1].
In the interval [0, x
], f (x) 0 and in the interval [
x, 1], f (x) 0. Now we make two
sub-cases depending on at which endpoint the maximum occurs.
(3i) Suppose 1/4 a 1/2. Then
f (0) f (1). So minimum is at x
, maximum is
at x = 0. f (
x) = a a + a
= 2 a a. So the differencebetween maximum and
2
minimum
is
1
+
a

2
a
=
(1

to 1 and so
2
a) . This is smallest when a is3 closest
2
1
(1 a) (1 1/ 2) = 3/2 2. This is bigger than 1/12 since ( 2 12 ) = 17/12 and
172 = 289 2 122 .
(3ii) Suppose 1/2 a
1. Nowf (1) f (0).
1 and minimum is at x
. The
Max is at
1 2

difference is a + 1/2 a + a a = 2a 2 a + 1/2 = ( 2a 2 ) . By a calculation


similar to the above it is bigger than 1/12.

6. Define fk (n) to be the sum of all possible products of k distinct integers chosen from
the set {1, 2, . . . , n}, i.e.,
fk (n) =

i1 i2 . . . ik .
1i1 <i2 <...<ik n

a) For k > 1, write a recursive formula for the function fk , i.e., a formula for fk (n) in
terms of f (m), where < k or ( = k and m < n).
b) Show that fk (n), as a function of n, is a polynomial of degree 2k.
c) Express f2 (n) as a polynomial in variable n.
Answer: a) Break up the terms in the definition of fk (n) into two groups: the terms in
which ik = n add up to nfk1 (n 1) and the remaining terms, i.e., the ones in which
ik n 1, add up to fk (n 1). So we get fk (n) = nfk1 (n 1) + fk (n 1).
c) By part a we have f2 (n) f2 (n 1) = nf1 (n 1) = n n(n1)
= 12 (n3 n2 ). Similarly
2
f2 (n 1) f2 (n 2) = 21 ((n 1)3 (n 1)2 ) and so on up to f2 (2) f2 (1) = 12 (23 22 ).
Note that f2 (1) = 0, which we may also write as 12 (13 12 ). Adding up, we get for
j=n

any n 1, f2 (n) = j=1 12 (j 3 j 2 ) = 12 ( n (n+1)


n(n+1)(2n+1)
), where we have used
4
6
standard formulas for the sum of first n cubes and of first n squares.
8

b) We prove the statement by induction on k. First f1 (n) = i=1 i = n(n+1)


, a polynomial
2
of degree 2 as desired. For k > 1, we have by part a the equation fk (n) fk (n 1) =
nfk1 (n 1). The right hand side is a polynomial of degree 1 + 2(k 1) = 2k 1, where
2(k 1) is the degree of fk1 (n 1) by induction and the added 1 comes from the factor
n. Since successive differences in the values of fk are given by a polynomial of degree
2k 1, the function fk on positive integers is given by a polynomial of degree 1 more, i.e.,
of degree 2k.
Note: The previous statement is a standard fact, which can be explained as follows. (1)
If we assume that fk (n) is a polynomial, then its degree is easily found, because for any
polynomial f of degree m, its successive difference function f (x)f (x1) is a polynomial
of degree m 1. (Reason: If the leading term of f (x) is axm , then the leading term in
f (x) f (x 1) is amxm1 , as seen by expanding the power of x 1 in axm a(x 1)m .
The remaining terms in f (x) f (x 1) do not matter because by expanding powers of
x 1 in them and simplifying, we only get monomials of degree < m 1.) (2) In fact,
based on the difference equation, fk (n) must be a polynomial in the variable n. This is a
consequence of the following well-known fact.
Claim: given a polynomial h(x) of degree d, there is a polynomial g(x) of degree d + 1
such that g(x) g(x 1) = h(x). Proof: Induction on d, the degree of h. If h(x) = c,
a constant, then g(x) = cx works. Now for d > 1, it is enough to find a polynomial g(x)

has degree < d, by


such that g(x) g(x 1) = xd (because if h(x) = cxd + h(x),
where h
and then cg(x) + g(x) works for h(x)). To find such g(x), notice
induction we find g for h
d+1
that for g1 (x) = x , we have h1 (x) = g1 (x) g1 (x 1) = (d + 1)xd + h2 (x), where h2 (x)
is a polynomial of degree d 1. By induction h2 (x) = g2 (x) g2 (x 1) for a polynomial
1
(g1 (x) g2 (x)) works.
g2 (x) of degree d. Now g(x) = d+1

Test Codes: UGA (Multiple-choice Type) and


UGB (Short Answer Type) 2013
Questions will be set on the following and related topics.
Algebra: Sets, operations on sets. Prime numbers, factorization of integers
and divisibility. Rational and irrational numbers. Permutations and combinations, Binomial Theorem. Logarithms. Polynomials: relations between
roots and coefficients, Remainder Theorem, Theory of quadratic equations
and expressions. Arithmetic and geometric progressions. Inequalities involving arithmetic, geometric & harmonic means. Complex numbers.
Geometry: Plane geometry. Geometry of 2 dimensions with Cartesian and
polar coordinates. Equation of a line, angle between two lines, distance from
a point to a line. Concept of a Locus. Area of a triangle. Equations of circle,
parabola, ellipse and hyperbola and equations of their tangents and normals.
Mensuration.
Trigonometry: Measures of angles. Trigonometric and inverse trigonometric functions. Trigonometric identities including addition formulae, solutions
of trigonometric equations. Properties of triangles. Heights and distances.
Calculus: Sequences - bounded sequences, monotone sequences, limit of a
sequence. Functions, one-one functions, onto functions. Limits and continuity. Derivatives and methods of differentiation. Slope of a curve. Tangents
and normals. Maxima and minima. Using calculus to sketch graphs of functions. Methods of integration, definite and indefinite integrals, evaluation of
area using integrals.

Reference (For more sample questions)


Test of Mathematics at the 10 + 2 level, Indian Statistical Institute. Published by Affiliated East-West Press Pvt. Ltd., 105, Nirmal Tower, 26
Barakhamba Road, New Delhi 110001.

Sample Questions for UGA


Instructions. UGA is a multiple choice examination. In each of the following questions, exactly one of the choices is correct. You get four marks
for each correct answer, one mark for each unanswered question, and zero
marks for each incorrect answer.

1 Define an = (12 + 22 + . . . + n2 )n and bn = nn (n!)2 . Recall n! is the


product of the first n natural numbers. Then,
(A) an < bn for all n > 1
(C) an = bn for infinitely many n

(B) an > bn for all n > 1


(D) None of the above

2 The sum of all distinct four digit numbers that can be formed using
the digits 1, 2, 3, 4, and 5, each digit appearing at most once, is
(A) 399900

(B) 399960

(C) 390000

(D) 360000

3 The last digit of (2004)5 is


(A) 4

(B) 8

(C) 6

(D) 2

4 The coefficient of a3 b4 c5 in the expansion of (bc + ca + ab)6 is


(A)

12!
3!4!5!

(B)

6
3!
3

(C) 33

(D) 3

6
3

5 Let ABCD be a unit square. Four points E, F , G and H are chosen


on the sides AB, BC, CD and DA respectively. The lengths of the
sides of the quadrilateral EF GH are , , and . Which of the
following is always true?

(A) 1 2 + 2 + 2 + 2 2 2

(B) 2 2 2 + 2 + 2 + 2 4 2
(C) 2 2 + 2 + 2 + 2 4

(D) 2 2 + 2 + 2 + 2 2 + 2
6 If log10 x = 10log100 4 then x equals
(A) 410

(B) 100

(C) log10 4

(D) none of the above

7 z1 , z2 are two complex numbers with z2 = 0 and z1 = z2 and satisfying


z1
z1 + z2
= 1. Then
is
z1 z2
z2
1

(A) real and negative


(B) real and positive
(C) purely imaginary
(D) none of the above need to be true always
8 The set of all real numbers x satisfying the inequality x3 (x+1)(x2)
0 is
(A) the interval [2, )
(B) the interval [0, )
(C) the interval [1, )
(D) none of the above
9 Let z be a non-zero complex number such that
nary. Then
(A) z is neither real nor purely imaginary
(C) z is purely imaginary

z
1+z

is purely imagi-

(B) z is real
(D) none of the above

10 Let A be the fixed point (0, 4) and B be a moving point (2t, 0). Let
M be the mid-point of AB and let the perpendicular bisector of AB
meet the y-axis at R. The locus of the mid-point P of M R is
(A) y + x2 = 2
(C) (y 2)2 x2 = 1/4

(B) x2 + (y 2)2 = 1/4


(D) none of the above

11 The sides of a triangle are given to be x2 + x + 1, 2x + 1 and x2 1.


Then the largest of the three angles of the triangle is
x
(A) 75
(B)
radians
(C) 120
(D) 135
x+1
12 Two poles, AB of length two metres and CD of length twenty metres are erected vertically with bases at B and D. The two poles
are at a distance not less than twenty metres. It is observed that
tan ACB = 2/77. The distance between the two poles is
(A) 72m

(B) 68m

(C) 24m

(D) 24.27m

13 If A, B, C are the angles of a triangle and sin2 A + sin2 B = sin2 C,


then C is equal to
(A) 30

(B) 90

(C) 45

(D) none of the above

14 In the interval (2, 0), the function f (x) = sin

1
x3

(A) never changes sign


(B) changes sign only once
(C) changes sign more than once, but finitely many times
(D) changes sign infinitely many times
15 The limit

(ex 1) tan2 x
x0
x3
lim

(A) does not exist


(C) exists and equals 2/3

(B) exists and equals 0


(D) exists and equals 1

16 Let f1 (x) = ex , f2 (x) = ef1 (x) and generally fn+1 (x) = efn (x) for all
d
n 1. For any fixed n, the value of
fn (x) is equal to
dx
(A) fn (x)
(B) fn (x)fn1 (x)
(C) fn (x)fn1 (x) f1 (x)
(D) fn+1 (x)fn (x) f1 (x)ex
17 If the function
x2 2x+A
sin x

f (x) =

if x = 0
if x = 0

is continuous at x = 0, then
(A) A = 0, B = 0
(C) A = 1, B = 1

(B) A = 0, B = 2
(D) A = 1, B = 0

18 A truck is to be driven 300 kilometres (kms.) on a highway at a constant speed of x kms. per hour. Speed rules of the highway require
that 30 x 60. The fuel costs ten rupees per litre and is consumed
at the rate 2 + (x2 /600) litres per hour. The wages of the driver are
200 rupees per hour. The most economical speed (in kms. per hour)
to drive the truck is

(A) 30
(B) 60
(C) 30 3.3
(D) 20 33
1

19 If b =
(A)

0
a
be

et
dt then
t+1
(B)

a1
a
be

et
dt is
ta1

(C) bea

(D) bea

20 In the triangle ABC, the angle BAC is a root of the equation

3 cos x + sin x = 1/2.


Then the triangle ABC is
3

(A) obtuse angled


(C) acute angled but not equilateral

(B) right angled


(D) equilateral

21 Let n be a positive integer. Consider a square S of side 2n units. Divide S into 4n2 unit squares by drawing 2n 1 horizontal and 2n 1
vertical lines one unit apart. A circle of diameter 2n 1 is drawn with
its centre at the intersection of the two diagonals of the square S. How
many of these unit squares contain a portion of the circumference of
the circle?
(A) 4n 2

(B) 4n

(C) 8n 4

(D) 8n 2

22 A lantern is placed on the ground 100 feet away from a wall. A man
six feet tall is walking at a speed of 10 feet/second from the lantern
to the nearest point on the wall. When he is midway between the
lantern and the wall, the rate of change (in ft./sec.) in the length of
his shadow is
(A) 2.4
(B) 3
(C) 3.6
(D) 12
23 An isosceles triangle with base 6 cms. and base angles 30 each is
inscribed in a circle. A second circle touches the first circle and also
touches the base of the triangle at its midpoint. If the second circle is
situated outside the triangle, then its radius (in cms.) is

(B) 3/2
(C) 3
(D) 4/ 3
(A) 3 3/2
24 Let n be a positive integer. Define
n+1

f (x) = min{|x 1|, |x 2|, . . . , |x n|}.


f (x)dx equals

Then
0

(A)

(n + 4)
4

(B)

(n + 3)
4

(C)

(n + 2)
2

(D)

(n + 2)
4

25 Let S = {1, 2, . . . , n}. The number of possible pairs of the form (A, B)
with A B for subsets A and B of S is
n
n
n
(A) 2n
(B) 3n
(C)
(D) n!
k
nk
k=0

26 The number of maps f from the set {1, 2, 3} into the set {1, 2, 3, 4, 5}
such that f (i) f (j) whenever i < j is
(A) 60

(B) 50

(C) 35

(D) 30

27 Consider three boxes, each containing 10 balls labelled 1, 2, . . . , 10.


Suppose one ball is drawn from each of the boxes. Denote by ni , the
4

label of the ball drawn from the i-th box, i = 1, 2, 3. Then the number
of ways in which the balls can be chosen such that n1 < n2 < n3 is
(A) 120
(B) 130
(C) 150
(D) 160
28 Let a be a real number. The number of distinct solutions (x, y) of the
system of equations (x a)2 + y 2 = 1 and x2 = y 2 , can only be
(A) 0, 1, 2, 3, 4 or 5
(C) 0, 1, 2 or 4

(B) 0, 1 or 3
(D) 0, 2, 3, or 4

29 The maximum of the areas of the isosceles triangles with base on the
positive x-axis and which lie below the curve y = ex is:
(A) 1/e
(B) 1
(C) 1/2
(D) e
30 Suppose a, b and n are positive integers, all greater than one. If an +bn
is prime, what can you say about n?
(A) The integer n must be 2
(B) The integer n need not be 2, but must be a power of 2
(C) The integer n need not be a power of 2, but must be even
(D) None of the above is necessarily true
31 Water falls from a tap of circular cross section at the rate of 2 metres/sec and fills up a hemispherical bowl of inner diameter 0.9 metres.
If the inner diameter of the tap is 0.01 metres, then the time needed
to fill the bowl is
(A) 40.5 minutes
(B) 81 minutes
(C) 60.75 minutes

(D) 20.25 minutes

32 The value of the integral


5/2
/2

equals (A) 1

etan
etan

1 (sin x)

1 (sin x)

(B)

+ etan

1 (cos x)

(C) e

dx
(D) none of these

33 The set of all solutions of the equation cos 2 = sin + cos is given
by
(A) = 0
(B) = n + 2 , where n is any integer
(C) = 2n or = 2n 2 or = n 4 , where n is any integer
(D) = 2n or = n + 4 , where n is any integer
34 For k 1, the value of

n
n+1
n+2
n+k
+
+
+ +
0
1
2
k

equals
5

n+k
n+1

(A)

n+k+1
n+k

(B) (n + k + 1)

(C)

n+k+1
n+1

(D)

n+k+1
n

+ cos1

2
+ sec1
3

35 The value of
sin1 cot sin1

1
2

5
6

8
3

is
(A) 0

(B) /6

(C) /4

(D) /2

36 Which of the following graphs represents the function

eu

f (x) =

2 /x

du,

for

x > 0 and

f (0) = 0?

(A)

(B)

(C)

(D)

37 If an =

1
1+ 2
n

22
1+ 2
n

32
1+ 2
n

lim a1/n
n

n2
1 + 2
n

, then

is
(A) 0

(B) 1

(C) e

(D)

e/2

38 The function x( x) is strictly increasing on the interval 0 < x < 1


if and only if
(A) 2
(C) < 1

(B) < 2
(D) > 2

39 Consider a circle with centre O. Two chords AB and CD extended


intersect at a point P outside the circle. If AOC = 43 and BP D =
18 , then the value of BOD is
(A) 36
(B) 29
(C) 7
(D) 25
6

40 A box contains 10 red cards numbered 1, . . . , 10 and 10 black cards


numbered 1, . . . , 10. In how many ways can we choose 10 out of the
20 cards so that there are exactly 3 matches, where a match means a
red card and a black card with the same number?
10 7
10 7 4
2
(B)
(A)
4
3
4
3
(C)

10 7
2
3

(D)

10
3

14
4

41 Let P be a point on the ellipse x2 + 4y 2 = 4 which does not lie on the


axes. If the normal at the point P intersects the major and minor axes
at C and D respectively, then the ratio P C : P D equals
(A) 2
(B) 1/2
(C) 4
(D) 1/4
42 The set of complex numbers z satisfying the equation
(3 + 7i)z + (10 2i)
z + 100 = 0
represents, in the complex plane,
(A) a straight line
(B) a pair of intersecting straight lines
(C) a pair of distinct parallel straight lines
(D) a point
43 The number of triplets (a, b, c) of integers such that a < b < c and
a, b, c are sides of a triangle with perimeter 21 is
(A) 7
(B) 8
(C) 11
(D) 12.
44 Suppose a, b and c are three numbers in G.P. If the equations
ax2 + 2bx + c = 0 and dx2 + 2ex + f = 0 have a common root, then
f
d e
, and are in
a b
c
(A) A.P.
(B) G.P.
(C) H.P.
(D) none of the above.
45 The number of solutions of the equation sin1 x = 2 tan1 x is
(A) 1
(B) 2
(C) 3
(D) 5.
46 Suppose ABCD is a quadrilateral such that BAC = 50 , CAD =
60 , CBD = 30 and BDC = 25 . If E is the point of intersection
of AC and BD, then the value of AEB is
(A) 75
(B) 85
(C) 95
(D) 110 .
47 Let R be the set of all real numbers. The function f : R R defined
by f (x) = x3 3x2 + 6x 5 is
(A) one-to-one, but not onto
(B) one-to-one and onto
7

(C) onto, but not one-to-one


(D) neither one-to-one nor onto.
48 Let L be the point (t, 2) and M be a point on the y-axis such that LM
has slope t. Then the locus of the midpoint of LM , as t varies over
all real values, is
(A) y = 2 + 2x2

(B) y = 1 + x2

(C) y = 2 2x2

(D) y = 1 x2 .

49 Suppose x, y (0, /2) and x = y. Which of the following statement


is true?
(A) 2 sin(x + y) < sin 2x + sin 2y for all x, y.
(B) 2 sin(x + y) > sin 2x + sin 2y for all x, y.
(C) There exist x, y such that 2 sin(x + y) = sin 2x + sin 2y.
(D) None of the above.
50 A triangle ABC has a fixed base BC. If AB : AC = 1 : 2, then the
locus of the vertex A is
(A) a circle whose centre is the midpoint of BC
(B) a circle whose centre is on the line BC but not the midpoint of
BC
(C) a straight line
(D) none of the above.
51 Let P be a variable point on a circle C and Q be a fixed point outside
C. If R is the mid-point of the line segment P Q, then the locus of R
is
(A) a circle
(B) an ellipse
(C) a line segment
(D) segment of a parabola
52 N is a 50 digit number. All the digits except the 26th from the right
are 1. If N is divisible by 13, then the unknown digit is
(A) 1

(B) 3

(C) 7

(D) 9.

53 Suppose a < b. The maximum value of the integral


b
a

3
x x2
4

over all possible values of a and b is


3
4
(A)
(B)
4
3
54 For any n 5, the value of 1 +

dx

(C)

3
2

(D)

1
1 1
+ + + n
lies between
2 3
2 1

2
.
3

n
and n
2
(D) none of the above.

n
2
(C) n and 2n

(B)

(A) 0 and

55 Let denote a cube root of unity which is not equal to 1. Then the
number of distinct elements in the set
is
(A) 4

(1 + + 2 + + n )m : m, n = 1, 2, 3,
(B) 5

(C) 7

(D) infinite.

56 The value of the integral


3
2

dx
loge x

(A) is less than 2

(B) is equal to 2

(C) lies in the interval (2, 3)

(D) is greater than 3.

57 The area of the region bounded by the straight lines x = 12 and x = 2,


and the curves given by the equations y = loge x and y = 2x is

(A) log1 2 (4 + 2) 25 loge 2 + 32


(B) log1 2 (4 2) 25 loge 2
e
e

(D) none of the above


(C) log1 2 (4 2) 25 loge 2 + 32
e

58 In a win-or-lose game, the winner gets 2 points whereas the loser gets
0. Six players A, B, C, D, E and F play each other in a preliminary
round from which the top three players move to the final round. After
each player has played four games, A has 6 points, B has 8 points and
C has 4 points. It is also known that E won against F. In the next set
of games D, E and F win their games against A, B and C respectively.
If A, B and D move to the final round, the final scores of E and F are,
respectively,
(A) 4 and 2
(B) 2 and 4
(C) 2 and 2
(D) 4 and 4.
59 The number of ways in which one can select six distinct integers from
the set {1, 2, 3, , 49}, such that no two consecutive integers are selected, is
49
48
43
(A)
5
(B)
6
5
6
(C)

25
6

(D)

44
.
6

60 Let n 3 be an integer. Assume that inside a big circle, exactly n


small circles of radius r can be drawn so that each small circle touches
the big circle and also touches both its adjacent small circles. Then,
the radius of the big circle is
9

(A) r cosec n

(C) r(1 + cosec 2n


)

(B) r(1 + cosec 2


n )
(D) r(1 + cosec n )

61 If n is a positive integer such that 8n + 1 is a perfect square, then


(A)
(B)
(C)
(D)

n must be odd
n cannot be a perfect square
2n cannot be a perfect square
none of the above

62 Let C denote the set of all complex numbers. Define


A = {(z, w) |z, w C and |z| = |w|}

B = {(z, w) |z, w C, and z 2 = w2 }.


Then,
(A) A = B
(C) B A and B = A

(B) A B and A = B
(D) none of the above

63 Let f (x) = a0 +a1 |x|+a2 |x|2 +a3 |x|3 , where a0 , a1 , a2 , a3 are constants.
Then
(A) f (x) is differentiable at x = 0 whatever be a0 , a1 , a2 , a3
(B) f (x) is not differentiable at x = 0 whatever be a0 , a1 , a2 , a3
(C) f (x) is differentiable at x = 0 only if a1 = 0
(D) f (x) is differentiable at x = 0 only if a1 = 0, a3 = 0
64 If f (x) = cos(x) 1 +

x2
2 ,

then

(A) f (x) is an increasing function on the real line


(B) f (x) is a decreasing function on the real line
(C) f (x) is increasing on < x 0 and decreasing on 0 x <
(D) f (x) is decreasing on < x 0 and increasing on 0 x <

65 The number of roots of the equation x2 + sin2 x = 1 in the closed


interval [0, 2 ] is
(A) 0

(B) 1

(C) 2

(D) 3

66 The set of values of m for which mx2 6mx+5m+1 > 0 for all real x is
(A) m < 41
(B) m 0
1
(D) 0 m < 14
(C) 0 m 4
67 The digit in the units place of the number 1! + 2! + 3! + . . . + 99! is
10

(A) 3

(B) 0
13 +23 +...+n3
n4
n
(B) 14

68 The value of lim


(A)

3
4

(D) 7

(C) 1

(D) 4

is:

69 For any integer n 1, define an =


(A)
(B)
(C)
(D)

(C) 1

1000n
n! .

Then the sequence {an }

does not have a maximum


attains maximum at exactly one value of n
attains maximum at exactly two values of n
attains maximum for infinitely many values of n

70 The equation x3 y + xy 3 + xy = 0 represents


(A) a circle
(B) a circle and a pair of straight lines
(C) a rectangular hyperbola (D) a pair of straight lines
71 For each positive integer n, define a function fn on [0, 1] as follows:

0
if
x=0

sin
if
0<x

2n
n

1
2
2

if
<x
sin

2n
n
n

3
2
3
fn (x) =
sin
if
<x

2n
n
n

.
.
.

..
..
..

sin n if n 1 < x 1.
2n
n
1

Then, the value of lim

n 0

(A)

fn (x) dx is

(B) 1

(C)

(D)

2
.

72 Let d1 , d2 , . . . , dk be all the factors of a positive integer n including 1


and n. If d1 + d2 + . . . + dk = 72, then d11 + d12 + + d1k is:
(A)

k2
72

(B)

72
k

(C)

72
n

(D) none of the above

73 A subset W of the set of real numbers is called a ring if it contains 1


and if for all a, b W , the numbers a b and ab are also in W . Let
S = 2mn | m, n integers and T = pq | p, q integers, q odd . Then
11

(A) neither S nor T is a ring


(C) T is a ring S is not a ring

12

(B) S is a ring T is not a ring


(D) both S and T are rings

Hints and Answers to selected problems.


There are also other ways to solve the problems apart from the ones sketched
in the hints. Indeed, a student should feel encouraged upon finding a different
way to solve some of these problems.
Hints and Answers to selected UGA Sample Questions.
1 (B). Take the nth root of an and bn and use A.M. G.M.
3 (A). As 2004 = 2000 +4, the last digits of (2004)5 and 45 are equal.
4 (D) Use binomial expansion of (bc + a (b + c))6 .
6 (B) Let y = log10 x. Then log10 y = log100 4. Hence y = 2.
8 (D) Check for test points.
9 (A) Check (B) and (C) are false, and then that (A) is true.
1
14 (D) sin x13 changes sign at the points (n) 3 for all n 1.
x

tan x
15 (D) Observe that (e 1)
= (e x1) sinx2 x cos12 x .
x3
16 (C) Use induction and chain rule of differentiation.
22 (B) Show that the height function is 60
t .
26 (C) Compute the number of maps such that f (3) = 5, f (3) = 4 etc..
Alternatively, define g : {1, 2, 3} {1, 2, . . . , 7} by g (i) = f (i) + (i 1).
Then, g is a strictly increasing function and its image is a subset of size 3 of
{1, 2, . . . 7}.
28 (D) Draw graphs of (x + y)(x y) = 0 and (x a)2 + y 2 = 1.
38 (A) Differentiate.
51 (A) Compute for C = x2 + y 2 = 1 and Q = (a, 0) for some a > 1.
2

57 (C) Compute the integral


1/2

2x dx

log xdx.
1/2

60 (D) Let s be distance between the centre of the big circle and the centre
of (any) one of the small circles. Then there exists a right angle triangle
with hypoteneuse s, side r and angle n .
61(C) If 8n + 1 = m2 , then 2n is a product of two consecutive integers.
62 (C) z 2 = w2 z = w B A. But |i| = 1 and i2 = 1.
63 (C) Amongst 1, |x|, |x|2 , |x|3 , only |x| is not differentiable at 0.
64 (D) Look at the derivative of f .
65 (B) Draw graphs of y = cos x and y = x and find the number of points
of intersections.
66 (D) Calculate the discriminant (b2 4ac) of the given quadratic.
67 (A) The unit digit of all numbers n! with n 5 is 0.
n

68 (B) Use the formula for

i3 .

i=1

69
70
72
73

(C) Find out the first values of n for which an+1


an becomes < 1.
2
2
(D) The equation is xy(x + y + 1) = 0.
(C) Multiply the given sum by n.
(D) Verify using the given definition of a ring.
13

Sample Questions for UGB


Instructions UGB consists of questions that will require you to provide
answers with appropriate justification.
1 Find the sum of all distinct four digit numbers that can be formed
using the digits 1, 2, 3, 4, 5, each digit appearing at most once.
2 How many natural numbers less than 108 are there, with sum of digits
equal to 7?
3 Consider the squares of an 8 8 chessboard filled with the numbers 1
to 64 as in the figure below. If we choose 8 squares with the property
that there is exactly one from each row and exactly one from each
column, and add up the numbers in the chosen squares, show that the
sum obtained is always 260.
1 2 3 4 5 6 7 8
9 10 11 12 13 14 15 16
17 18 19 20 21 22 23 24
25 26 27 28 29 30 31 32
33 34 35 36 37 38 39 40
41 42 43 44 45 46 47 48
49 50 51 52 53 54 55 56
57 58 59 60 61 62 63 64
4 Consider the function
loge (2 + x) x2n sin x
n
1 + x2n
defined for x > 0. Is f (x) continuous at x = 1? Justify your answer.
Show that f (x) does not vanish anywhere in the interval 0 x 2 .
Indicate the points where f (x) changes sign.
f (x) = lim

5 An isosceles triangle with base 6 cms. and base angles 30o each is
inscribed in a circle. A second circle, which is situated outside the
triangle, touches the first circle and also touches the base of the triangle
at its midpoint. Find its radius.
6 Suppose a is a complex number such that
1
1
a2 + a + + 2 + 1 = 0.
a a
If m is a positive integer, find the value of
1
1
a2m + am + m + 2m .
a
a
7 Let an = 1 . . . 1 with 3n digits. Prove that an is divisible by 3an1 .
8 Let f (u) be a continuous function and, for any real number u, let [u]
denote the greatest integer less than or equal to u. Show that for any
1

x > 1,
[x]

[u]([u] + 1)f (u)du = 2


1

f (u)du.

i
i=1

9 If a circle intersects the hyperbola y = 1/x at four distinct points


(xi , yi ), i = 1, 2, 3, 4, then prove that x1 x2 = y3 y4 .
10 Two intersecting circles are said to be orthogonal to each other if the
tangents to the two circles at any point of intersection are perpendicular to each other. Show that every circle through the points (2, 0) and
(2, 0) is orthogonal to the circle x2 + y 2 5x + 4 = 0.
11 Show that the function f (x) defined below attains a unique minimum
for x > 0. What is the minimum value of the function? What is the
value of x at which the minimum is attained?
1
1
f (x) = x2 + x + + 2 for x = 0.
x x
Sketch on plain paper the graph of this function.
12 Show that there is exactly one value of x which satisfies the equation
2 cos2 (x3 + x) = 2x + 2x .
13 Let S = {1, 2, . . . , n}. Find the number of unordered pairs {A, B} of
subsets of S such that A and B are disjoint, where A or B or both
may be empty.
14 An oil-pipe has to connect the oil-well O and the factory F , between
which there is a river whose banks are parallel. The pipe must cross
the river perpendicular to the banks. Find the position and nature of
the shortest such pipe and justify your answer.
15 Find the maximum value of x2 + y 2 in the bounded region, including
the boundary, enclosed by y = x2 , y = x2 and x = y 2 + 1.
16 Let x = (x1 , . . . , xn ) and y = (y1 , . . . , yn ) where x1 , , xn , y1 , , yn
are real numbers. We write x > y if either x1 > y1 or for some k,
with 1 k n 1, we have x1 = y1 , . . . , xk = yk , but xk+1 > yk+1 .
Show that for u = (u1 , . . . , un ), v = (v1 , . . . , vn ), w = (w1 , . . . , wn ) and
z = (z1 , . . . , zn ), if u > v and w > z, then u + w > v + z.
17 How many real roots does x4 + 12x 5 have?
18 For any positive integer n, let f (n) be the remainder obtained on
dividing n by 9. For example, f (263) = 2.
(a) Let n be a three-digit number and m be the sum of its digits.
Show that f (m) = f (n).
2

(b) Show that f (n1 n2 ) = f (f (n1 ) f (n2 )) where n1 , n2 are any two
positive three-digit integers.
19 Find the maximum among 1, 21/2 , 31/3 , 41/4 , . . ..
20 Show that it is not possible to have a triangle with sides a, b and c
whose medians have lengths 23 a, 32 b and 45 c.
21 For real numbers x, y and z, show that
|x| + |y| + |z| |x + y z| + |y + z x| + |z + x y|.
22 Let
P (x) = xn + an1 xn1 + an2 xn2 + + a1 x + a0

be a polynomial with integer coefficients, such that P (0) and P (1) are
odd integers. Show that:
(a) P (x) does not have any even integer as root.
(b) P (x) does not have any odd integer as root.
23 Let N = {1, 2, . . . , n} be a set of elements called voters. Let C =
{S : S N } be the set of all subsets of N . Members of C are called
coalitions. Let f be a function from C to {0, 1}. A coalition S N
is said to be winning if f (S) = 1; it is said to be a losing coalition
if f (S) = 0. A pair N, f as above is called a voting game if the
following conditions hold.
(a) N is a winning coalition.
(b) The empty set is a losing coalition.
(c) If S is a winning coalition and S S , then S is also winning.
(d) If both S and S are winning coalitions, then S S = , i.e., S
and S have a common voter.
Show that the maximum number of winning coalitions of a voting
game is 2n1 . Find a voting game for which the number of winning
coalitions is 2n1 .
24 Suppose f is a real-valued differentiable function defined on [1, ) with
f (1) = 1. Suppose, moreover, that f satisfies f (x) = 1/(x2 + f 2 (x)).
Show that f (x) 1 + /4 for every x 1.
25 If the normal to the curve x2/3 + y 2/3 = a2/3 at some point makes an
angle with the X-axis, show that the equation of the normal is
y cos x sin = a cos 2.
26 Suppose that a is an irrational number.
(a) If there is a real number b such that both (a+b) and ab are rational
numbers, show that a isa quadratic
surd. (a is a quadratic surd
if it is of the form r + s or r s for some rationals r and s,
where s is not the square of a rational number).
3

(b) Show that there are two real numbers b1 and b2 such that
(i) a + b1 is rational but ab1 is irrational.
(ii) a + b2 is irrational but ab2 is rational.
(Hint: Consider the two cases, where a is a quadratic surd
and a is not a quadratic surd, separately).
27 Let A, B, and C be three points on a circle of radius 1.
(a) Show that the area of the triangle ABC equals
1
(sin(2ABC) + sin(2BCA) + sin(2CAB)) .
2
(b) Suppose that the magnitude of ABC is fixed. Then show that
the area of the triangle ABC is maximized when BCA = CAB.
(c) Hence or otherwise show that the area of the triangle ABC is
maximum when the triangle is equilateral.
28 In the given figure, E is the midpoint of the arc
ABEC and ED is perpendicular to the chord BC
at D. If the length of the chord AB is l1 , and that of
BD is l2 , determine the length of DC in terms of l1
and l2

E
C
B

D
A

29 (a) Let f (x) = x xe1/x , x > 0. Show that f (x) is an increasing


function on (0, ), and limx f (x) = 1.
(b) Using part (a) and calculus, sketch the graphs of y = x 1, y = x,
y = x + 1, and y = xe1/|x| for < x < using the same X
and Y axes.
30 For any integer n greater than 1, show that
2n <

2n
n

<

2n
n1
i=0

i
(1 )
n

31 Show that there exists a positive real number x = 2 such that log2 x =
x
2 . Hence obtain the set of real numbers c such that
log2 x
=c
x
has only one real solution.
32 Find a four digit number M such that the number N = 4 M has the
following properties.
(a) N is also a four digit number.
(b) N has the same digits as in M but in the reverse order.
33 Consider a function f on nonnegative integers such that f (0) = 1,
f (1) = 0 and f (n) + f (n 1) = nf (n 1) + (n 1)f (n 2) for n 2.
4

Show that
f (n)
=
n!

n
k=0

(1)k
.
k!

34 Of all triangles with a given perimeter, find the triangle with the maximum area. Justify your answer.
35 A 40 feet high screen is put on a vertical wall 10 feet above your eyelevel. How far should you stand to maximize the angle subtended by
the screen (from top to bottom) at your eye?
36 Study the derivatives of the function
y=

x3 4x

and sketch its graph on the real line.

37 Suppose P and Q are the centres of two disjoint circles C1 and C2


respectively, such that P lies outside C2 and Q lies outside C1 . Two
tangents are drawn from the point P to the circle C2 , which intersect
the circle C1 at points A and B. Similarly, two tangents are drawn
from the point Q to the circle C1 , which intersect the circle C2 at
points M and N . Show that AB = M N .
38 Evaluate: lim

2n
1
.
log
n
2n

39 Consider the equation x5 + x = 10. Show that


(a) the equation has only one real root;
(b) this root lies between 1 and 2;
(c) this root must be irrational.
40 In how many ways can you divide the set of eight numbers {2, 3, . . . , 9}
into 4 pairs such that no pair of numbers has g.c.d. equal to 2?
41 Suppose S is the set of all positive integers. For a, b S, define
l.c.m(a, b)
ab=
g.c.d(a, b)
For example, 8 12 = 6.
Show that exactly two of the following three properties are satisfied :
(a) If a, b S then a b S.
(b) (a b) c = a (b c) for all a, b, c S.
(c) There exists an element i S such that a i = a for all a S.

Hints and Answers to selected problems.


There are also other ways to solve the problems apart from the ones sketched
in the hints. Indeed, a student should feel encouraged upon finding a different
way to solve some of these problems.
Hints and Answers to selected UGB Sample Questions.
1. The answer is 399960. For each x {1, 2, 3, 4, 5}, there are 4! such
numbers whose last digit is x. Thus the digits in the unit place of all the
120 numbers add up to 4! (1 + 2 + 3 + 4 + 5). Similarly the numbers at tens
place add up to 360 and so on. Thus the sum is 360 (1 + 10 + 100 + 1000).
3. Let the chosen entries be in the positions (i, ai ), 1 i 8. Thus a1 , . . . , a8
is a permutation of {1, . . . , 8}. The entry in the square corresponding to
8

(i, j)th place is i + 8 (j 1). Hence the required sum is


5. Radius is

3 3
2 .

i=1

(i + 8 (aj 1)).

Use trigonometry.
n

7. Observe that an = an1 1 + t + t2 where t = 103

9. Substitute y = x1 in the equation of a circle and clear denominator to


get a degree 4 equation in x. The product of its roots is the constant term,
which is 1.
11. The function f (x) 4 is a sum of squares and hence non-negative. So
the minimum is 4 which is attained at x = 1.
n

13. The number is 3 2+1 . An ordered pair (A, B) of disjoint subsets of S is


determined by 3 choices for every element of S (either it is in A, or in B or
in neither of them). Hence such pairs are 3n in number. An unordered pair
will be counted twice in this way, except for the case A and B both empty.
n
Hence the number is 1 + 3 21 .
15. Answer is 5. The maximum is attained at points (2, 1) and (2, 1).
17. Answer is 2. Let f be the given polynomial. Then f (0) is negative and
f is positive as x tends to .
Hence it has at least 2 real roots. Since the
derivative of f is zero only at 3 3, it cannot have more than two real roots.

1
3
19. Maximum
is
3. Either check the maximum of the function x x , or

compare 3 3 with n n.
21. Rewrite the given inequality in terms of the new variables = x + y z,
= y + z x, = x + z y, and use the triangle inequality.

A Model Question Paper for B.Math/B.Stat

Test Code : UGA

Booklet No.

Forenoon

Questions : 30

Time : 2 hours

Write your Name, Registration Number, Test Centre, Test Code and the
Number of this Booklet in the appropriate places on the Answersheet.

This test contains 30 questions in all. For each of the 30 questions, there
are four suggested answers. Only one of the suggested answers is correct.
You will have to identify the correct answer in order to get full credit for
that question. Indicate your choice of the correct answer by darkening the
appropriate oval completely on the answersheet.

You will get


4 marks for each correctly answered question,
0 marks for each incorrectly answered question and
1 mark for each unattempted question.

All rough work must be done on this booklet only.


You are not allowed to use calculator.

WAIT FOR THE SIGNAL TO START.

Each of the following questions has exactly one correct answer among the
given options and you have to identify it.
1. A rod AB of length 3 rests on a wall as follows:

P is a point on AB such that AP : P B = 1 : 2. If the rod slides along


the wall, then the locus of P lies on
(A) 2x + y + xy = 2
(B) 4x2 + y 2 = 4
(C) 4x2 + xy + y 2 = 4
(D) x2 + y 2 x 2y = 0.
2. Consider the equation x2 + y 2 = 2007. How many solutions (x, y) exist
such that x and y are positive integers?
(A) None
(B) Exactly two
(C) More than two but finitely many
(D) Infinitely many.
3. Consider the functions f1 (x) = x, f2 (x) = 2 + loge x, x > 0 (where e is
the base of natural logarithm). The graphs of the functions intersect
(A) once in (0, 1) and never in (1, )
(B) once in (0, 1) and once in (e2 , )
(C) once in (0, 1) and once in (e, e2 )
(D) more than twice in (0, ).

4. Consider the sequence


n

un =
r=1

r
, n 1.
2r

Then the limit of un as n is


(A) 1

(B) 2

(C) e

(D) 1/2.

5. Suppose that z is any complex number which is not equal to any of


{3, 3, 3 2 } where is a complex cube root of unity. Then
1
1
1
+
+
z 3 z 3 z 3 2
equals
(A)

3z 2 +3z
(z3)3

(B)

3z 2 +3z
z 3 27

(C)

3z 2
z 3 3z 2 +9z27

(D)

3z 2
.
z 3 27

6. Consider all functions f : {1, 2, 3, 4} {1, 2, 3, 4} which are one-one,


onto and satisfy the following property:
if f (k) is odd then f (k + 1) is even, k = 1, 2, 3.
The number of such functions is
(A) 4

(B) 8

(C) 12

(D) 16.

7. A function f : R R is defined by
1

f (x) =

e x ,

x>0

x 0.

Then
(A) f is not continuous
(B) f is differentiable but f is not continuous
(C) f is continuous but f (0) does not exist
(D) f is differentiable and f is continuous.
8. The last digit of 9! + 39966 is
(A) 3

(B) 9

(C) 7
2

(D) 1.

9. Consider the function


f (x) =

2x2 + 3x + 1
, 2 x 3.
2x 1

Then
(A) maximum of f is attained inside the interval (2, 3)
(B) minimum of f is 28/5
(C) maximum of f is 28/5
(D) f is a decreasing function in (2, 3).
10. A particle P moves in the plane in such a way that the angle between
the two tangents drawn from P to the curve y 2 = 4ax is always 90 .
The locus of P is
(A) a parabola

(B) a circle

(C) an ellipse

(D) a straight line.

11. Let f : R R be given by


f (x) = |x2 1|, x R.
Then
(A) f has a local minima at x = 1 but no local maximum
(B) f has a local maximum at x = 0 but no local minima
(C) f has a local minima at x = 1 and a local maximum at x = 0
(D) none of the above is true.
12. The number of triples (a, b, c) of positive integers satisfying
2a 5b 7c = 1
is
(A) infinite

(B) 2

(C) 1

(D) 0.

13. Let a be a fixed real number greater than 1. The locus of z C


satisfying |z ia| = Im(z) + 1 is
(A) parabola

(B) ellipse

(C) hyperbola
3

(D) not a conic.

14. Which of the following is closest to the graph of tan(sin x), x > 0?

15. Consider the function f : R \ {1} R \ {2} given by


f (x) =

2x
.
x1

Then
(A) f is one-one but not onto
(B) f is onto but not one-one
(C) f is neither one-one nor onto
(D) f is both one-one and onto.
16. Consider a real valued continuous function f satisfying f (x+1) = f (x)
for all x R. Let
t

f (x) dx, t R.

g(t) =
0

Define h(t) = limn

g(t+n)
n ,

provided the limit exists. Then

(A) h(t) is defined only for t = 0


(B) h(t) is defined only when t is an integer
(C) h(t) is defined for all t R and is independent of t
(D) none of the above is true.
4

17. Consider the sequence a1 = 241/3 , an+1 = (an + 24)1/3 , n 1. Then


the integer part of a100 equals
(A) 2

(B) 10

(C) 100

(D) 24.

18. Let x, y (2, 2) and xy = 1. Then the minimum value of


4
9
+
4 x2 9 y 2
is
(A) 8/5

(B) 12/5

19. What is the limit of

(C) 12/7

1
1+ 2
n +n

(D) 15/7.

n2 + n

as n ?
(A) e

(B) 1

(D) .

(C) 0

20. Consider the function f (x) = x4 + x2 + x 1, x (, ). The


function
(A) is zero at x = 1, but is increasing near x = 1
(B) has a zero in (, 1)
(C) has two zeros in (1, 0)
(D) has exactly one local minimum in (1, 0).
21. Consider a sequence of 10 As and 8 Bs placed in a row. By a run we
mean one or more letters of the same type placed side by side. Here is
an arrangement of 10 As and 8 Bs which contains 4 runs of A and 4
runs of B:
AAAB B AB B B AAB AAAAB B
In how many ways can 10 As and 8 Bs be arranged in a row so that
there are 4 runs of A and 4 runs of B?
(A) 2

9
3

7
3

(B)

9
3

7
3

(C)

10
4

8
4

(D)

10
5

8
5

22. Suppose n 2 is a fixed positive integer and


f (x) = xn |x|, x R.
Then
(A) f is differentiable everywhere only when n is even
(B) f is differentiable everywhere except at 0 if n is odd
(C) f is differentiable everywhere
(D) none of the above is true.
23. The line 2x + 3y k = 0 with k > 0 cuts the x axis and y axis at points
A and B respectively. Then the equation of the circle having AB as
diameter is
(A) x2 + y 2 k2 x k3 y = k 2
(B) x2 + y 2 k3 x k2 y = k 2
(C) x2 + y 2 k2 x k3 y = 0
(D) x2 + y 2 k3 x k2 y = 0.
24. Let > 0 and consider the sequence
xn =

( + 1)n + ( 1)n
, n = 1, 2, . . . .
(2)n

Then limn xn is
(A) 0 for any > 0
(B) 1 for any > 0
(C) 0 or 1 depending on what > 0 is
(D) 0, 1 or depending on what > 0 is.
25. If 0 < < /2 then
(A) < sin
(B) cos(sin ) < cos
(C) sin(cos ) < cos(sin )
(D) cos < sin(cos ).
6

26. Consider a cardboard box in the shape of a prism as shown below. The
length of the prism is 5. The two triangular faces ABC and A B C are
congruent and isosceles with side lengths 2,2,3. The shortest distance
between B and A along the surface of the prism is

(A)

29

(B)

28

(C)

29

(D)

29

27. Assume the following inequalities for positive integer k:

1
1

< k+1 k < .


2 k+1
2 k
The integer part of
9999
k=2

equals
(A) 198

(B) 197

(C) 196

(D) 195.

28. Consider the sets defined by the inequalities


A = {(x, y) R2 : x4 + y 2 1}, B = {(x, y) R2 : x6 + y 4 1}.
Then
(A) B A
(B) A B
(C) each of the sets A B, B A and A B is non-empty
(D) none of the above is true.

29. The number

210
11

11

is
10 2 10 2 10 2
1
2
3
10 2 10 2 10 2
strictly larger than 1
2
3
10 2 10 2 10 2 10 2 10
1
2
3
4
5
2 10 2 10
less than or equal to 10
1
2
3
2
2
2
2
10
10
10
10
equal to 10
1
2
3
4
5 .

(A) strictly larger than


(B)

(C)
(D)

10 2 10
4
5
10 2
but
4

strictly smaller than

2 10 2
4

30. If the roots of the equation x4 + ax3 + bx2 + cx + d = 0 are in geometric


progression then
(A) b2 = ac

(B) a2 = b

(C) a2 b2 = c2

(D) c2 = a2 d.

A Model Question Paper for B.Math/B.Stat


BOOKLET No.

TEST CODE : UGB


Afternoon Session

There are 3 pages in this booklet.


The exam has 8 questions.
Answer as many as you can.

Time : 2 hours

Write your Name, Registration number, Test Centre, Test Code and the
Number of this booklet in the appropriate places on the answer-booklet.

ALL ROUGH WORK IS TO BE DONE ON THIS BOOKLET


AND/OR THE ANSWER-BOOKLET.
CALCULATORS ARE NOT ALLOWED.

STOP! WAIT FOR THE SIGNAL TO START.

P.T.O.
1

1. Let X, Y , Z be the angles of a triangle.


(i) Prove that
tan

X
Y
X
Z
Z
Y
tan + tan tan + tan tan = 1.
2
2
2
2
2
2

(ii) Using (i) or otherwise prove that


tan

X
Y
Z
1
tan tan .
2
2
2
3 3

2. Let be a real number. Consider the function


2

g(x) = ( + | x |)2 e(5|x|) , < x < .


(i) Determine the values of for which g is continuous at all x.
(ii) Determine the values of for which g is differentiable at all x.

3. Write the set of all positive integers in a triangular array as


1
2
4
7
11

3
5
8
12
.

6
9
13
.
.

10
14
.
.
.

15
.
.
.
.

.
.
.
.
.

.
.
.
.
.

Find the row number and column number where 20096 occurs. For
example 8 appears in the third row and second column.

4. Show that the polynomial x8 x7 + x2 x + 15 has no real root.


5. Let m be a natural number with digits consisting entirely of 6s and
0s. Prove that m is not the square of a natural number.
P.T.O.
2

6. Let 0 < a < b.


(i) Show that amongst the triangles with base a and perimeter a + b,
the maximum area is obtained when the other two sides have
equal length 2b .
(ii) Using the result of (i) or otherwise show that amongst the quadrilateral of given perimeter the square has maximum area.

7. Let 0 < a < b. Consider two circles with radii a and b and centers
(a, 0) and (b, 0) respectively with 0 < a < b. Let c be the center of any
circle in the crescent shaped region M between the two circles and
tangent to both (See figure below). Determine the locus of c as its
circle traverses through region M maintaining tangency.

c01
0

00
11
1
0
00
0 11
1

a b

8. Let n 1, S = {1, 2, . . . , n}. For a function f : S S, a subset D S


is said to be invariant under f , if f (x) D for all x D. Note that
the empty set and S are invariant for all f . Let deg(f ) be the number
of subsets of S invariant under f .
(i) Show that there is a function f : S S such that deg(f ) = 2.

(ii) Further show that for any k such that 1 k n there is a


function f : S S such that deg(f ) = 2k .

You might also like